Тест по Клинической фармакологии система подготовки к тестам Gee Test oldkyx com Список вопросов по Клинической фармакологии. Предэкзаменационный тест по фармакологии. Тесты по фармакологии Тест по фармакологии 90 вопросов

Тест по Клинической фармакологии

система подготовки к тестам Gee Test oldkyx.com

Список вопросов по Клинической фармакологии

1. Период полувыведения лекарства - это:
1) [-] время достижения максимальной концентрации лекарства в плазме;

2) [-] время, в течение которого лекарство достигает системного кровотока;

3) [-] время, в течение которого лекарство распределяется в организме;

4) [+] время, за которое концентрация лекарства в плазме снижает¬ся на 50%;

5) [-] время, за которое половина введенной дозы достигает орга¬на-мишени.

2. Широта терапевтического действия - это:
1) [-] терапевтическая доза лекарства;

2) [-] отношение концентрации лекарства в органе или ткани к концентрации его в плазме крови;

3) [+] диапазон между минимальной терапевтической и минималь¬ной токсической концентрациями лекарства в плазме;

4) [-] процент не связанного с белком лекарства;

5) [-] диапазон между минимальной и максимальной терапевтиче¬скими концентрациями лекарства.

3. К рецепторным средствам конкурентного действия относятся:
1) [-] НПВС (нестероидные противовоспалительные средства);

2) [+] β-адреноблокаторы;

3) [-] петлевые диуретики;

4) [-] нитраты;

5) [-] фторхинолоны.

4. Функцию печени и почек следует учитывать при назначении следующих лекарственных средств:
1) [-] липофильных, образующих неактивные метаболиты;

2) [+] липофильных, образующих активные метаболиты;

3) [-] гидрофильных;

4) [-] гепатотоксичных;

5) [-] нефротоксичных.

5. Селективность действия лекарственного вещества зависит от:
1) [-] периода полувыведения;

2) [-] способа приема;

3) [-] связи с белком;

4) [-] объема распределения;

5) [+] дозы.

6. Для кинетики насыщения характерно:
1) [+] увеличение периода полувыведения введенной дозы при не¬измененном клиренсе;

2) [-] скорость элиминации пропорциональна концентрации препа¬рата в плазме и дозе;

3) [-] период полувыведения не пропорционален введенной дозе.

7. Фактор, определяющий необходимость перерасчета режима введения препарата при ХПН:
1) [-] высокая липофильность препарата;

2) [-] низкая связь с белками плазмы;

3) [-] наличие систем активного канальцевого пути экскреции;

4) [+] высокая степень экскреции в неизмененном виде.

8. Какие препараты с большей легкостью проходят через ГЭБ?
1) [-] с высокой растворимостью в воде;

2) [+] с высокой растворимостью в в жирах;

3) [-] проявляющие свойства слабых кислот;

4) [-] проявляющие свойства слабых оснований;

5) [-] со слабой связью с белками плазмы.

9. В каком случае происходит более полное всасывание?
1) [-] всасывание из желудка препарата, проявляющего свойства слабого основания;

2) [-] всасывание из тонкого кишечника препарата, проявляющего свойства слабой кислоты;

3) [+] всасывание из тонкого кишечника препарата, проявляющего свойства слабого основания.

10. В понятие «пресистемный метаболизм» входит:
1) [+] биотрансформация препаратов в печени при первом прохож¬дении и в кишечнике;

2) [-] биотрансформация препаратов в кишечнике;

3) [-] биотрансформация препаратов в печени при первом прохож¬дении и в почках;

4) [-] биотрансформация препаратов в печени, в почках и в кишечнике.

11. Строго дозозависимой является следующая группа побочных эффектов:
1) [-] фармацевтические;

2) [-] фармакогенетические;

3) [-] аллергические;

4) [-] мутагенные;

5) [+] синдром отмены.

12. Определите группу препаратов с узким терапевтическим индексом:
1) [-] β-блокаторы;

2) [-] пенициллины;

3) [+] сердечные гликозиды;

4) [-] ингибиторы АПФ;

5) [-] мощные диуретики.

13. Проведение лекарственного мониторинга желательно при лечении следующей группой препаратов:
1) [+] противосудорожными;

2) [-] β2-симптомомиметиками;

3) [-] пенициллинами;

4) [-] глюкокортикоидами;

5) [-] М-холинолитиками.

14. К отсроченным относится следующая группа побочных эффектов:
1) [-] токсические;

2) [-] развитие лекарственной зависимости;

3) [-] фармакогенетические;

4) [+] канцерогенные;

5) [-] синдром отмены.

15. Развитие асистолии возможно при комбинации пропранолола с:
1) [-] фенобарбиталом;

2) [-] фуросемидом;

3) [+] верапамилом;

4) [-] фенитоином;

5) [-] ранитидином.

16. Риск токсических эффектов увеличивается при комбинации гентамицина с:
1) [+] фуросемидом;

2) [-] пенициллином;

3) [-] метилксантинами;

4) [-] макролидами;

5) [-] глюкокортикоидами.

17. Риск развития нежелательной беременности увеличивается при сочетании оральных контрацептивов с:
1) [-] гипотензивными;

2) [-] витамином С;

3) [-] алкоголем;

4) [+] тетрациклином;

5) [-] глюкокортикоидами.

18. При патологии почек возникают следующие изменения фармакокинетики лекарств, кроме:
1) [-] нарушения почечной экскреции;

2) [-] увеличения концентрации лекарств в плазме крови;

3) [-] уменьшения связывания с белками плазмы;

4) [-] увеличения Т1/2;

5) [+] уменьшения биодоступности.

19. Цирроз печени вызывают следующие изменения фармакокинетики лекарств, кроме:
1) [-] снижения пресистемного метаболизма;

3) [-] увеличения Т1/2;

4) [-] увеличения биодоступности;

5) [+] уменьшения объема распределения.

20. При сердечной недостаточности наблюдаются следующие изменения фармакокинетики дигоксина, кроме:
1) [-] снижения абсорбции в ЖКТ на 30%;

2) [-] уменьшения связывания с белками плазмы;

3) [+] усиления метаболизма в печени;

4) [-] снижения почечной экскреции;

5) [-] увеличения Т1/2.

21. Алкоголь при однократном приеме больших доз приводит к:
1) [-] увеличению абсорбции лекарств;

3) [+] замедлению метаболизма в печени;

4) [-] снижению почечной экскреции;

5) [-] увеличению Т1/2.

22. Никотин приводит к:
1) [-] уменьшению абсорбции лекарств;

2) [-] увеличению объема распределения лекарств;

3) [-] уменьшению связи с белком плазмы;

4) [+] усилению метаболизма в печени;

5) [-] усилению почечной экскреции лекарств.

23. Обычной формой высвобождения лекарственного вещества характеризуется:
1) [-] нитронг;

2) [-] сустак-мите;

3) [+] нитросорбид;

4) [-] нифедипин-GITS;

5) [-] верапамил SR.

24. Для купирования приступа стенокардии применяют сублингвально таблетированную лекарственную форму:
1) [-] нитронг;

2) [-] сустак;

3) [+] нитросорбид;

4) [-] атенонолол;

5) [-] верапамил SR.

25. Для предотвращения развития толерантности при регулярном применении нитратов безнитратный интервал должен составлять:
1) [-] 2-4 часа;

2) [-] 4-6 часов;

3) [-] 6-8 часов;

4) [+] 8-12 часов.

26. Для усиления антиангинального эффекта наиболее безопасным является сочетание:
1) [-] верапамил + пропранолол;

2) [-] верапамил + атенолол;

3) [-] верапамил + метопролол;

4) [+] верапамил + изосорбида динитрат;

5) [-] верапамил + дилтиазем.

27. Методами оценки антиангинальной эффективности препарата являются все перечисленные, кроме:
1) [-] холтеровского мониторирования ЭКГ;

2) [+] мониторирования суточного АД;

3) [-] стресс-Эхо;

4) [-] тредмил-теста;

5) [-] ВЭМ-пробы.

28. У больного стенокардией в сочетании с артериальной гипертонией имеют преимущество препараты:
1) [-] нитраты;

29. При стенокардии напряжения препаратами выбора являются препараты следующего класса:

2) [+] блокаторы β-адренорецепторов;

3) [-] блокаторы α-адренорецепторов;

4) [-] агонисты имидазолиновых рецепторов;

5) [-] блокаторы рецепторов ангиотензина II.

30. При вазоспастической стенокардии препаратами выбора являются препараты следующего класса:
1) [-] блокаторы гистаминовых рецепторов;

2) [-] блокаторы β-адренорецепторов;

3) [-] блокаторы α-адренорецепторов;

4) [+] блокаторы кальциевых каналов;

5) [-] блокаторы рецепторов ангиотензина II.

31. Адекватным методом контроля за эффективностью и безопасностью гипотензивной терапии является:
1) [-] суточное мониторирование ЭКГ;

2) [+] суточное мониторирование АД;

3) [-] разовые измерения АД;

4) [-] измерение показателей ФВД;

5) [-] динамика интервала QT на ЭКГ.

32. Выберите нежелательный эффект, не характерный для верапамила:
1) [-] брадикардия;

2) [-] запоры;

3) [-] развитие AV-блокады;

4) [-] отеки голеней и стоп;

5) [+] бронхоспазм.

33. Для лечения артериальной гипертензии препаратом первого выбора у больного хронической сердечной недостаточностью является:
1) [+] эналаприл;

2) [-] верапамил;

3) [-] клофелин;

4) [-] празозин;

5) [-] нифедипин.

34. Укажите гипотензивный препарат, увеличивающий активность симпатоадреналовой системы:
1) [+] нифедипин;

2) [-] клофелин;

3) [-] каптоприл;

4) [-] метопролол;

5) [-] ирбесартан.

35. При артериальной гипертонии в сочетании с синусовой тахикардией предпочтение следует отдать:
1) [-] блокаторам кальциевых каналов производных дигидропиридина;

2) [-] петлевым диуретикам;

3) [+] β-адреноблокаторам;

4) [-] α-адреноблокаторам;

5) [-] тиазидным диуретикам.

36. α1-адреноблокаторы являются препаратами выбора для лечения артериальной гипертонии:
1) [-] у пациентов с заболеваниями печени;

2) [-] у больных с нарушениями ритма;

3) [+] у пожилых мужчин с аденомой предстательной железы и за¬трудненным мочеиспусканием;

4) [-] у пациентов со стенокардией;

5) [-] у больных с инфарктом миокарда в анамнезе.

37. Для лечения артериальной гипертонии у больных с бронхиальной астмой не могут быть использованы:
1) [-] блокаторы кальциевых каналов;

2) [-] антагонисты рецепторов к ангиотензину II;

3) [-] α1-адреноблокаторы;

4) [+]β-адреноблокаторы;

5) [-] диуретики.

38. Пациентам с артериальной гипертонией и инфарктом миокарда в анамнезе в первую очередь следует назначить:
1) [+] β-адреноблокаторы;

2) [-] диуретики;

3) [-] блокаторы кальциевых каналов;

4) [-] агонисты имидазолиновых рецепторов;

5) [-] α1-адреноблокаторы.

39. Препаратами первого выбора у пациентов с артериальной гипертонией и выраженным стенозом периферических артерий являются:
1) [-] неселективные β-адреноблокаторы;

2) [+] антагонисты кальция;

3) [-] диуретики;

4) [-] блокаторы рецепторов к ангиотензину II;

5) [-] агонисты α2-адренорецепторов.

40. Перечислите группы препаратов, улучшающих прогноз пациентов с ХСН:
1) [-] ингибиторы АПФ;

2) [-] β-блокаторы;

3) [-] блокаторы рецепторов к ангиотензину II;

4) [-] спиронолактон;

41. Перечислите препараты, обладающие прямым положительным инотропным эффектом:
1) [-] дигоксин;

2) [-] допамин;

3) [-] амринон;

4) [-] левосимендан;

5) [+] все перечисленные препараты.

42. Перечислите β-блокаторы, обладающие доказанной эффективностью в лечении пациентов с ХСН:
1) [-] атенолол;

2) [-] пропранолол;

3) [+] карведилол;

4) [-] соталол;

43. Укажите показания к назначению спиронолактона:
1) [-] уровень калия в крови > 5, 5 ммоль/л;

2) [-] отеки голеней и стоп;

3) [+] сердечная недостаточность IV ФК по классификации NYHA;

4) [-] клиренс креатинина менее 30 мл/мин;

44. Титрование дозы ингибиторов АПФ и β-блокаторов у пациентов с ХСН подразумевает:
1) [-] начало терапии с минимальной дозы препарата;

2) [-] увеличение дозы препарата каждые 2 недели;

3) [-] достижение целевой дозы препарата;

4) [-] снижение количества госпитализаций и увеличение продол¬жительности жизни больного;

5) [+] все перечисленные признаки.

45. Показания для назначения амлодипина при ХСН:
1) [-] застойная сердечная недостаточность;

2) [+] неконтролируемые цифры артериального давления;

3) [-] инфаркт миокарда в анамнезе;

4) [-] нарушения ритма;

5) [-] все перечисленные признаки.

46. Принципы диуретической терапии при ХСН:
1) [-] назначение диуретиков при сердечной недостаточности II-IV ФК по классификации NYHA;

2) [-] снижение веса на 0,5-1,0 кг в сутки;

3) [-] контроль артериального давления;

4) [-] контроль за уровнем калия в крови;

5) [+] все перечисленные.

47. Укажите предпочтительный путь введения лекарственных препаратов при застойной сердечной недостаточности:
1) [-] сублингвальный;

2) [-] ректальный;

3) [+] парентеральный;

4) [-] пероральный;

5) [-] все перечисленные пути введения.

48. Эффекты ингибитора АПФ у пациентов с ХСН:
1) [-] влияние ингибитора АПФ на смертность зависит от продол¬жительности лечения;

2) [-] снижение риска летального исхода более выражено у паци¬ентов с более высоким ФК;

3) [-] наличие дозозависимого эффекта ингибитора АПФ у боль¬ных сердечной недостаточностью;

4) [+] все перечисленные.

49. Перечислите препараты, увеличивающие продолжительность потенциала действия:
1) [-] хинидин;

2) [-] прокаинамид;

3) [-] амиодарон;

4) [-] дигоксин;

5) [+] все перечисленные препараты.

50. Препараты, удлиняющие интервал QT:
1) [-] клиндамицин;

2) [-] амиодарон;

3) [-] котримоксазол;

4) [-] хинидин;

5) [+] все перечисленные препараты.

51. Препараты, удлиняющие интервал PQ:
1) [-] лидокаин;

2) [-] хинидин;

3) [-] дизопирамид;

4) [+] дигоксин;

5) [-] все перечисленные препараты к назначению антиаритмических препаратов.

52. Показаниями к назначению антиаритмических препаратов являются:
1) [-] частые нарушения ритма;

2) [-] желудочковые экстрасистолы - 6 в минуту;

3) [-] нарушение ритма высоких градаций;

4) [+] нарушение гемодинамики;

5) [-] все перечисленные.

53. Перечислите препараты, имеющие холинолитические побочные эффекты:
1) [-] амиодарон;

2) [-] веропамил;

3) [-] лидокаин;

4) [+] хинидин;

5) [-] все перечисленные препараты.

54. Перечислите препараты, повышающие порог фибрилляции:
1) [-] кордарон;

2) [-] бретилий тозилат;

3) [-] соталол;

4) [-] пропранолол;

5) [+] все перечисленные препараты.

55. Перечислите препараты, назначаемые для профилактики нарушений ритма при WPW-синдроме:
1) [-] дигоксин;

2) [-] дилтиазем;

3) [+] амиодарон;

4) [-] прокаинамид;

5) [-] все перечисленные препараты.

56. Показания к восстановлению ритма при постоянной форме мерцательной аритмии:
1) [-] частые эпизоды тахисистолии;

2) [-] слабость синусового узла;

3) [+] наличие тромбоэмболии в анамнезе;

4) [-] неэффективность проводимой медикаментозной терапии;

5) [-] все перечисленные.

57. Препарат выбора для лечения пароксизмов желудочковой тахикардии:
1) [-] лидокаин;

2) [-] прокаинамид;

3) [-] бретилий;

4) [-] пропафенон;

5) [+] все перечисленные препараты.

58. Побочные эффекты амиодарона:
1) [-] фотосенсибилизация;

2) [-] нарушение функции щитовидной железы;

3) [-] сухой кашель;

4) [-] преходящее повышение активности печеночных аминотрансфераз;

5) [+] все перечисленные эффекты.

59. Показания к применению аденозина:
1) [-] пароксизм мерцательной аритмии;

2) [+] реципрокные наджелудочковые тахикардии;

3) [-] желудочковая тахикардия;

4) [-] экстрасистолия;

5) [-] все перечисленные.

60. Метод оценки эффективности антиаритмической терапии в амбулаторной практике:
1) [+] мониторинг ЭКГ по Холтеру;

3) [-] пробы с физической нагрузкой;

4) [-] ЭФИ (электорофизиологическое исследование);

5) [-] все перечисленные методы.

61. К ингаляционным глюкокортикоидам относится:
1) [-] гидрокортизон;

2) [+] беклометазон;

3) [-] преднизолон;

4) [-] полькорталон;

5) [-] дексаметазон.

62. К селективным β2-агонистам длительного действия относится:
1) [-] флутиказон;

2) [+] сальметерол;

3) [-] сальбутамол;

4) [-] фенотерол;

5) [-] тербуталин.

63. Для купирования приступа бронхиальной астмы применяется:
1) [-] тиотропиум бромид;

2) [-] теопэк;

3) [-] кромогликат натрия;

4) [+] сальбутамол;

5) [-] будесонид.

64. К холинолитикам длительного действия относят:
1) [-] ипратропиум бромид;

2) [-] кромгликат натрия;

3) [+] тиотропиум бромид;

4) [-] окситропиум бромид;

5) [-] триамцинолон ацетонид.

65. Побочным эффектом ингаляционных глюкокортикостероидов является:
1) [-] головная боль;

2) [-] ожирение;

3) [+] кандидоз полости рта;

4) [-] сахарный диабет;

5) [-] полиурия.

66. К муколитическим средствам относится:
1) [-] кодеин;

2) [-] кромогликат натрия;

3) [+] ацетилцистеин;

4) [-] сальметерол;

5) [-] теофиллин.

67. При одновременном применении повышает концентрацию теофиллина в крови:
1) [+] офлоксацин;

2) [-] пенициллин;

3) [-] цефтриаксон;

4) [-] гентамицин;

5) [-] бисептол.

68. При одновременном применении снижает концентрацию теофиллина в крови:
1) [-] пефлоксацин;

2) [-] циметидин;

3) [+] рифампицин;

4) [-] эритромицин;

5) [-] ампиокс.

69. К бронхолитикам не относятся:
1) [-] метилксантины;

2) [-] холинолитики;

3) [-] симпатомиметики;

4) [+] блокаторы лейкотриеновых рецепторов.

70. Препаратом базисной терапии при хроническом обструктивном бронхите является:
1) [+] тиотропиум бромид;

2) [-] недокромил натрия;

3) [-] фенотерол;

4) [-] монтелукаст;

5) [-] эуфиллин.

71. Тахикардия как побочный эффект развивается при приеме всех перечисленных препаратов, кроме:
1) [-] сальбутамола;

2) [-] изопротеринола;

3) [-] фенотерола;

4) [-] теофиллина;

5) [+] ипратропиума бромида.

72. К комбинированным средствам для лечения бронхиальной астмы не относят:
1) [-] дитек;

2) [-] серетид;

3) [-] симбикорт;

4) [+] беклазон;

5) [-] беродуал.

73. Наиболее выраженным противовоспалительным эффектом обладает ингаляционный глюкокортикостероид:
1) [-] бекламетазона дипропионат;

2) [-] будесонид;

3) [-] триамцинолона ацетонид;

4) [+] флутиказона пропионат;

5) [-] флунизолид.

74. Препаратом выбора при наличии инфекций, вызванных метициллинрезистентными штаммами золотистого стафилококка, является:
1) [-] азитромицин;

2) [-] метронидазол;

3) [-] гентамицин;

4) [+] линезолид;

5) [-] цефуроксим.

75. Выберите группу антибактериальных препаратов для лечения инфекций, вызванных внутриклеточными возбудителями:
1) [+] макролиды;

2) [-] пенициллины;

3) [-] аминогликозиды;

4) [-] цефалоспорины;

5) [-] сульфаниламиды.

76. Укажите группу антибактериальных препаратов, обладающих наибольшей антианаэробной активностью:
1) [-] гликопептиды;

2) [-] аминопенициллины;

3) [-] тетрациклины;

4) [-] аминогликозиды;

5) [+] нитроимидазолы.

77. Нефротоксичными являются все перечисленные антибактериальные препараты, кроме:
1) [-] гентамицина;

2) [-] карбенициллина;

3) [+] азитромицина;

4) [-] цефазолина;

5) [-] ванкомицина.

78. Укажите антибактериальный препарат, малоактивный в отношении пневмококка:
1) [-] азитромицин;

2) [-] пенициллин;

3) [-] цефтриаксон;

4) [+] ципрофлоксацин;

5) [-] левомицетин.

79. Выберите рациональную комбинацию антибактериальных препаратов, обладающую синергизмом действия в отношении грамположительных микроорганизмов и безопасностью:
1) [-] пенициллины + тетрациклины;

2) [-] пенициллины + цефалоспорины;

3) [-] аминогликозиды + гликопептиды;

4) [+] пенициллины + аминогликозиды;

5) [-] пенициллины + сульфаниламиды.

80. Хорошо проникают через гематоэнцефалический барьер следующие антибактериальные препараты:
1) [-] линкозамиды;

2) [-] макролиды;

3) [-] тетрациклины;

4) [-] аминогликозиды;

5) [+] цефалоспорины III генерации.

81. Препаратом выбора при крупозной пневмонии является:
1) [-] ципрофлоксацин;

2) [-] доксициклин;

3) [-] гентамицин;

4) [-] цефотаксим;

5) [+] бензилпенициллин.

82. Препаратом выбора при тонзиллофарингите является:
1) [+] цефуроксим аксетил;

2) [-] доксициклин;

3) [-] цефтазидим;

4) [-] офлоксацин;

5) [-] фурагин.

83. Препаратами выбора при инфекциях желчевыводящих путей являются:
1) [-] аминогликозиды;

2) [-] нитрофураны;

3) [+] цефалоспорины III поколения;

4) [-] макролиды;

5) [-] природные пенициллины.

84. При госпитальных инфекциях, возникших в реанимационных отделениях, препаратами выбора являются следующие комбинации антибактериальных препаратов:
1) [-] ампициллин + гентамицин;

2) [+] цефтазидим + амикацин;

3) [-] цефуроксим + эритромицин;

4) [-] клиндамицин + гентамицин;

5) [-] норфлоксацин + пенициллин.

85. Предпочтительной группой антибактериальных препаратов при лечении хронического простатита являются:
1) [-] сульфаниламиды;

2) [-] карбапенемы;

3) [-] хинолоны;

4) [-] линкозамины;

5) [+] фторхинолоны.

86. Выберите препарат, максимально подавляющий секрецию соляной кислоты:
1) [-] пирензепин;

2) [-] циметидин;

3) [-] мизопростол;

4) [-] антациды;

5) [+] омепразол.

87. Максимальное количество побочных эффектов среди Н2-блокаторов имеет:
1) [+] циметидин;

2) [-] роксатидин;

3) [-] низатидин;

4) [-] ранитидин;

5) [-] фамотидин.

88. Угнетает цитохром Р-450:
1) [-] омепразол;

2) [-] пирензепин;

3) [+] циметидин;

4) [-] фамотидин;

5) [-] лансопразол.

89. Синдром отдачи вызывают:
1) [-] синтетические простагландины;

2) [-] антациды;

4) [-] М-холинолитики;

5) [+] Н2-блокаторы.

90. При язвенной болезни двенадцатиперстной кишки рационально назначать антациды:
1) [-] до еды;

2) [-] во время еды;

3) [+] через 1, 5-2 часа после еды;

4) [-] через 5 часов после еды;

5) [-] вне зависимости от приема пищи.

91. Для профилактики язв, вызванных приемом НПВС, наиболее эффективны:
1) [-] антациды;

2) [-] Н2-блокаторы;

3) [-] блокаторы «протоновой помпы»;

4) [+] синтетические простагландины;

5) [-] М-холинолитики.

92. В эрадикационной терапии для уничтожения Н. pylori используется следующий антибиотик:
1) [-] карбенициллин;

2) [-] эритромицин;

3) [-] цефоперазон;

4) [+] кларитромицин;

5) [-] хлорамфеникол.

93. Большое практическое значение имеет развитие устойчивости Н. pylori к:
1) [-] ванкомицину;

2) [+] метронидазолу;

3) [-] тетрациклину;

4) [-] нитрофуранам;

5) [-] цефотаксиму.

94. Бактерицидным действием против Н. pylori обладает:
1) [-] сукральфат (вентер);

2) [+] субцитрат висмута (де-нол);

3) [-] альмагель;

4) [-] фамотидин;

5) [-] пирензепин.

95. В эрадикационной терапии для уничтожения Н. pylori используются:
1) [-] антациды;

2) [-] синтетические простагландины;

3) [+] блокаторы «протоновой помпы»;

4) [-] М-холинолитики;

5) [-] гликопептиды.

96. Кортикостероиды усиливают токсичность:
1) [-] теофиллина;

2) [+] тиазидных диуретиков;

3) [-] препаратов золота;

4) [-] противоязвенных средств.

97. Эффект глюкокортикоидов снижает:
1) [-] циметидин;

2) [-] аспирин;

3) [-] диклофенак;

4) [-] амиодарон;

5) [+] рифампицин.

98. Наиболее высокой минералокортикоидной активностью обладает:
1) [-] полькартолон;

2) [-] преднизолон;

3) [+] гидрокортизон;

4) [-] дексаметазон.

99. После приема НПВС быстрее развивается эффект:
1) [-] противовоспалительный;

2) [+] анальгетический;

3) [-] антикоагуляционный.

100. Наиболее выраженным анальгетическим свойством обладает:
1) [-] ацетилсалициловая кислота;

2) [-] ибупрофен;

3) [-] напроксен;

4) [+] парацетамол.

101. Механизмом развития ульцирогенного эффекта НПВС является:
1) [-] повышение кислотности желудочного сока;

2) [+] снижение синтеза простагландинов в слизистой оболочке же¬лудка;

3) [-] снижение репарации слизистой оболочки.

102. Наиболее выраженным противовоспалительным свойством обладает:
1) [+] фенилбутазон;

2) [-] метамизол;

3) [-] пироксикам;

4) [-] парацетамол;

5) [-] ибупрофен.

103. Ранним побочным эффектом глюкокортикоидов является:
1) [-] катаракта;

2) [-] миопатия;

3) [-] остеопороз;

4) [-] кушингоидный синдром;

5) [+] стероидный диабет.

104. Укажите эффект, не характерный для кортикостероидов:
1) [-] противовоспалительный;

2) [-] противоаллергический;

3) [-] противошоковый;

4) [-] иммунодепрессивный;

Волгоград 2011 год


ТЕСТЫ ПО КЛИНИЧЕСКОЙ ФАРМАКОЛОГИИ ДЛЯ СТУДЕНТОВ V КУРСА ПЕДИАТРИЧЕСКОГО ФАКУЛЬТЕТА

001. Равновесная концентрация препарата (Сss) это:

а) Концентрация препарата, при которой развивается максимальный терапевтический эффект.

б) Концентрация препарата, при которой развивается минимальный терапевтический эффект.

в) Установившаяся концентрация препарата при его повторном введении, при которой количество вводимого препарата равно количеству выводимого препарата.

г) Установившаяся концентрация препарата при его повторном введении, при которой появляются первые побочные эффекты препарата.

002. Элиминация вещества это:

а) Суммарный результат экскреции и метаболизма вещества.

б) Результат метаболизма вещества.

в) Результат экскреции вещества печенью.

г) Результат экскреции вещества почками.

003. Клиренс препарата это:

а) Процент снижения концентрации вещества в крови в единицу времени.

б) Количество вещества в процентах от принятой дозы, элиминируемое в течение суток.

в) Объём крови, который освобождается от лекарственного средства в единицу времени.

г) Скорость снижения концентрации вещества в единицу времени.

004. Абсорбция лекарственных средств не зависит от:

а) Активности микросомальных ферментов печени.

б) Пути введения ЛС.

в) Растворимости ЛС в воде и липидах.

г) Интенсивности кровотока в месте введения ЛС.

005. Величина фармакологического эффекта зависит от:

а) Концентрации лекарственного средства в биофазе.

б) Времени введения препарата.

в) Скорости введения препарата.

г) Концентрации лекарственного средства в жировой ткани.

006. Механизм действия ингибиторов АПФ реализуется через:

а) Влияние на активность ферментов.

в) Физико-химическое влияние на мембраны клеток.

007. Равновесная концентрация препарата (Сss) устанавливается через:

а) 1-2 периода полувыведения препарата.

б) 4-6 периодов полувыведения препарата.

в) 2-3 периода полувыведения препарата.

г) 7-9 периодов полувыведения препарата.

008. Тахифилаксия это:

а) Быстрое развитие толерантности после нескольких повторных приёмов ЛС.

б) Накопление ЛС в организме.

в) Быстрое выведение ЛС из организма.

г) Угнетение активности ферментных систем печени.

009. Укажите индуктор микросомального окисления:

а) Фенобарбитал.

б) Циметидин.

в) Амиодарон.

г) Ловастатин.

010. Укажите ингибиторы микросомального окисления:

а) Противосудорожные препараты.

б) Макролиды.

в) Противотуберкулёзные препараты (изониазид, рифампицин).

г) Алкоголь.

011. Повышение pH мочи ускоряет выведение:

а) Слабых кислот.

б) Слабых оснований.

012. Укажите препарат, для которого значимо возможное вытеснение из связи с альбуминами:

а) Варфарин.

б) Атенолол.

в) Гентамицин.

г) Будесонид.

013. Укажите ингибитор микросомального окисления в печени:

а) Грейпфрутовый сок.

б) Алкоголь.

в) Пища, приготовленная на углях.

г) Сигаретные смолы.

014. Терапевтическим лекарственным мониторингом называется:

а) Контроль за скоростью выведения препарата.

б) Контроль за правильностью введения препарата.

в) Контроль за равновесной концентрацией препарата.

г) Контроль за побочными эффектами препарата.

015. Метод, позволяющий случайно включать пациентов в оду из групп при проведении клинических исследований, называется:

а) метаанализ.

б) рандомизация.

в) стратификация.

г) сортировка.

016. К фазам клинических исследований не относится:

а) Исследования на животных.

б) Исследования на здоровых добровольцах.

в) Исследования на ограниченном числе больных.

г) Исследования на большом количестве больных

017. I фаза клинических исследований включает:

а) Исследования на здоровых добровольцах.

б) Исследования на ограниченном числе больных.

в) Исследования на большом количестве больных.

г) Исследования на животных.

018. Тератогенный эффект ЛС:

а) Формирование пороков развития у плода.

б) Гибель зародыша до установления беременности.

в) Развитие внутриутробной гипотрофии плода, функциональная незрелость органов.

019. Препараты, для которых обнаружено наличие риска для человека, однако польза от их применения во время беременности превышает возможный риск, относятся к категории:

020. У новорожденных объем распределения:

а) Для водорастворимых препаратов повышен, для жирорастворимых - понижен.

б) Для водорастворимых препаратов понижен, для жирорастворимых - повышен

в) Для водорастворимых препаратов понижен, для жирорастворимых - понижен

г) Для водорастворимых препаратов повышен, для жирорастворимых - повышен

021. Увеличение свободной фракции ЛС в крови новорожденного связано:

а) С меньшим количеством белков в плазме крови.

б) С низкой скоростью клубочковой фильтрации.

в) С небольшой массовой долей подкожно-жировой клетчатки с высоким процентом воды.

г) С небольшой мышечной массой.

022. Дженерический препарат фармакокинетически эквивалентен оригинальному если эквивалентны:

а) Максимальная концентрация, время достижения максимальной концентрации, площадь под кривой концентрация-время;

б) Максимальная концентрация, время достижения максимальной концентрации, период полувыведения.

в) Биодоступность, период полувыведения.

023. К методам фармакоэкономического анализа не относятся:

а) Метод затраты-эффективность.

б) Метод затраты-прибыль.

в) Метод затраты-полезность.

г) Метод затраты-выгода

024. Метод фармакоэкономического анализа, при котором все результаты представлены в денежном эквиваленте называется:

а) Минимизация затрат.

б) Затраты-полезность.

в) Затраты-выгода.

г) Затраты-эффективность.

025. Хронофармакология изучает:

а) Фармакокинетическое и фармакодинамическое взаимодействие ЛС с организмом в зависимости от биоритмов.

б) Активность ферментов печени в зависимости от биоритмов.

в) Влияние различных ЛС на биоритмы.

001. Целевым уровнем АД у детей с артериальной гипертензией является:

а) Менее 90-го перцентиля кривой распределения АД.

б) Менее 95-го перцентиля кривой распределения АД.

в) Менее 85-го перцентиля кривой распределения АД.

002. При вторичной артериальной гипертензии у детей медикаментозная терапия назначается:

а) Через 6 месяцев после неэффективности немедикаментозной терапии;

б) Через 12 месяцев после неэффективности немедикаментозной терапии;

в) Сразу после постановки диагноза;

г) Не назначается.

003. Суррогатным критерием эффективности терапии артериальной гипертензии является:

а) Снижение систолического давления на 10 мм.рт.ст.;

б) Достижение целевого уровня АД;

в) Снижение систолического и диастолического давления на10 мм.рт.ст.;

004. Нерациональной комбинацией гипотензивных препаратов является:

а) Диуретик + бета-блокатор.

б) Антагонист кальция + диуретик.

в) Антагонист кальция + бета-блокатор.

г) Бета– блокатор + ИАПФ.

005. Нефропротективным действием не обладают:

а) Бета-адреноблокаторы.

в) Антагонисты кальция.

г) Блокаторы рецепторов к ангиотензину II

006. При артериальной гипертензии с сопутствующей тахиаритмией препаратами выбора являются:

а) Бета-адреноблокаторы.

б) Диуретики.

г) Препараты центрального действия.

007. Препарат выбора для лечения артериальной гипертензии у беременных:

а) Клонидин.

б) Метилдофа.

в) Гуанфацин.

г) Моксонидин.

008. Выберете неселективные бета-блокаторы с вазодилатирующими свойствами:

а) Карведилол, лабеталол

б) Талинолол, ацебутолол

в) Атенолол, метопролол,

г) Пропранолол, надолол, соталол

009. Изменение ЧСС под действием бета-блокаторов:

а) Уменьшают ЧСС в покое (все) и при физической нагрузке (без ВСА).

б) Уменьшают ЧСС в покое (без ВСА) и при физической нагрузке (все).

в) Уменьшают ЧСС в покое (с ВСА) и при физической нагрузке (все).

г) Уменьшают ЧСС в покое (все) и при физической нагрузке (с ВСА).

010. Назначение больным с лабильным сахарным диабетом I типа опасно в связи с тем, что эти препараты

а) Маскируют симптомы гипогликемии.

б) Уменьшают высвобождение инсулина бета-клетками поджелудочной железы..

в) Тормозят мобилизацию глюкозы в печени в случае гипогликемии.

г) Снижают толерантность больных к глюкозе.

011. Среди всех антагонистов кальция единственным препаратом, блокирующим кальциевые каналы Т-типа, является:

а) Дилтиазем

б) Нифедипин

в) Мибефрадил.

г) Верапамил.

012. Дигидропиридины блокируют кальциевые каналы с преимущественным расположением:

а) В AV – узле.

б) В артериях.

в) И в артериях, и в AV – узле.

013. Механизм действия антагонистов кальция реализуется через:

а) Физико-химическое влияние на мембраны клеток.

б) Влияние на специфические рецепторы.

в) Влияние на активность ферментов.

г) Прямое химическое взаимодействие ЛС.

014. Антагонисты кальция дигидропиридинового ряда противопоказаны при:

а) Аортальном стенозе.

б) Митральном стенозе.

в) Аортальной недостаточности.

015. Изменения водно-электролитного баланса на фоне терапии ИАПФ:

а) Выведение калия, задержка натрия и воды.

б) Выведение калия, натрия и воды.

в) Выведение натрия и воды, задержка калия.

г) Выведение воды, задержка калия и натрия.

016. Выберите ИАПФ, который имеет два пути выведения:

а) Трандолаприл.

б) Квинаприл.

в) Фозиноприл.

г) Периндоприл.

017. Выберите группу препаратов, предпочтительную в лечении артериальной гипертензии на фоне выраженной почечной недостаточности:

б) Блокаторы рецепторов к ангиотензину II

в) Бета-блокаторы

г) Антагонисты кальция

018. Терапия ИАПФ сопровождается:

а) Повышением чувствительности периферических тканей к инсулину.

б) Снижением чувствительности периферических тканей к инсулину.

в) Ухудшением липидного профиля крови.

019. Преимуществом блокаторов АТ1 рецепторов перед ИАПФ является:

а) Возможно назначение при развитии ангионевротического отека на фоне лечения ИАПФ.

б) Возможно назначение при исходной гиперкалиемии.

в) Возможно назначение при стенозе почечных артерий.

020. Диуретики, используемые для длительной терапии артериальной гипертензии:

а) Петлевые.

б) Тиазидные.

в) Калийсберегающие.

г) Осмотические.

021. Ингибитором карбоангидразы является:

а) Этакриновая кислота.

б) Ацетазоламид.

в) Спиронолактон.

г) Индапамид.

022. Место действия тиазидных и тиазидоподобных диуретиков:

а) Проксимальные извитые канальцы.

б) Собирательные трубочки.

в) Нисходящий отдел петли Генле.

г) Восходящего отдела петли Генле и дистальные извитые канальцы.

023. Особенностью действия тиазидных и тиазидоподобных диуретиков является:

а) «Низкий терапевтический потолок» - хороший эффект на низких дозах.

б) Дозозависимый эффект – увеличение диуреза при увеличении дозы препарата.

в) Синдром отмены

г) Развитие гипокальциемии.

024. Среди всех тиазидных диуретиков особенностью препарата индапамида является:

а) Лучший профиль безопасности.

б) Более выраженный диуретический эффект.

в) Менее выраженный гипотензивный эффект.

г) Отрицательное влияние на углеводный обмен.

025. Особенностью фармакодинамики ацетазоламида является:

а) Сохранение ионов калия.

б) Выраженное диуретическое действие.

в) Дозозависимый эффект.

г) Снижение внутричерепной гипертензии.

026. При назначении петлевых диуретиков детям с непереносимостью сульфаниламидных производных рекомендуется выбрать:

а) Торасемид.

б) Фуросемид.

в) Этакриновую кислоту.

г) Буметанид.

001. Действие аденозина на проводимость в AV-узле:

а) Кратковременная блокада проведения.

б) Кратковременное улучшение проведения.

в) Не влияют.

002. Среди антиаритмических препаратов I класса фазу реполяризации укорачивают:

003. Препараты Ia класса действуют:

а) Только на предсердия.

б) Только на желудочки

в) На предсердия и желудочки

004. Отличительной особенностью препаратов Ia класса по сравнению с другими препаратами I класса является:

а) Максимально выраженный отрицательный инотропный эффект.

б) Максимальная по продолжительности блокада натриевых каналов.

в) Отсутствие ваголитической активности.

г) Наличие альфа-адреномиметической активности.

005. Препаратами выбора для лечения нарушений ритма, связанных с гиперсимпатикотонией являются:

а) Антагонисты кальция.

б) Мембраностабилизирующие средства.

в) Бета-адреноблокаторы.

г) Местные анестетики.

006. К антиаритмическим препаратам III класса относятся:

а) Блокаторы калиевых каналов

б) Блокаторы натриевых каналов

в) Антагонисты кальция

г) Бета - адреноблокаторы

007. Механизм действия сердечных гликозидов связан:

а) С блокадой натрий-калиевой АТФазы.

б) С активацией ДНК-гиразы.

в) С блокадой фосфолипазы А2.

г) С активацией циклооксигеназы.

008. Фармакодинамические эффекты дигоксина:

а) Положительный инотропный и батмотропный, отрицательный хронотропный и дромотропный.

б) Отрицательный инотропный и батмотропный, положительный хронотропный и дромотропный.

в) Положительный инотропный и хронотропный, отрицательный батмотропный и дромотропный.

г) Отрицательный инотропный и дромотропный, положительный хронотропный и батмотропный.

009. При длительной антиаритмической терапии появление серо-фиолетовой окраски кожи лица и рук, фотодерматита открытых участков кожи, аутоиммунного тиреоидита, интерстициального пневмонита характерно для:

а) Новокаинамида.

б) Соталола.

в) Амиодарона.

г) Пропафенона.

010. Препараты, угнетающие проведение импульса по дополнительным путям:

а) Ia, Ic, III классов.

б) Ib, Ic, II классов.

в) Ia, Ib, Ic классов

г) Ia, III, IV классов.

011. При длительной антиаритмической терапии развитие волчаночноподобного синдрома характерно для:

а) Новокаинамида.

б) Амиодарона.

в) Пропроналола.

г) Пропафенона.

012. Укажите препарат, обладающий фармакодинамическими эффектами всех четырех классов антиаритмиков:

а) Новокаинамид;

б) Лидокаин;

в) Амиодарон;

г) Аймалин

013. Препаратом выбора для лечения желудочковой тахикардии по типу "пируэт" является:

а) Магния сульфат.

б) Амиодарон.

в) Лидокаин.

г) Новокаинамид

014. Препаратом выбора для лечения желудочковой тахикардии является:

а) Лидокаин.

б) Аденозин.

в) Дигоксин.

г) Нифедипин.

015. При назначении комбинированной антиаритмической терапии препараты назначаются:

а) В половинных дозах.

б) В полных дозах.

в) В удвоенных дозах.

016. При врожденном синдроме удлиненного интервала QT с целью профилактики развития аритмий назначают:

а) Препараты класса Ia .

б) Препараты класса Ic.

в) Бета-блокаторы.

г) амиодарон.

017. Пациентам с WPW-синдромом противопоказано назначение:

а) Новокаинамида.

б) Пропафенона.

в) Верапамила.

г) Амиодарона.

018. Препаратом выбора в лечении полной AV-блокады является:

а) атропин.

б) магния сульфат.

в) аденозин.

г) дигоксин.

019. Препарат выбора для лечения дигиталисной интоксикации:

а) Новокаинамид.

б) Лидокаин.

в) Дифенин.

г) Верапамил.

020. При постоянной форме мерцательной аритмии для урежения частоты желудочковых ответов предпочтительно назначение:

а) Амиодарона.

б) Пропафенона.

в) Дигоксина.

г) Верапамила.

001. Механизм действия НПВС связан с блокадой фермента:

а) циклооксигеназа;

б) фосфодиэстераза;

в) 5-липооксигеназа;

г) фосфолипаза А2.

002. Выберите препарат, обладающий наибольшей селективностью в отношении ЦОГ-2:

а) аспирин;

б) ибупрофен;

в) целекоксиб;

г) индометацин;

003. Выберите препарат из группы НПВС не обладающий противовоспалительной активностью:

а) Парацетамол.

б) Ацетилсалициловая кислота.

в) Ибупрофен.

г) Нимесулид.

а) Ибупрофен.

б) Диклофенак.

в) Метамизол.

г) Ацетилсалициловая кислота.

005. Выберите правильный ответ. НПВС:

а) угнетают альтерацию и пролиферацию;

б) угнетают эксудацию и пролиферацию (некоторые препараты);

в) угнетают эксудацию и альтерацию;

г) угнетают все фазы воспаления;

006. Выберите группу препаратов, которая рекомендована к совместному применению с НПВС с целью снижения риска поражения ЖКТ:

а) синтетический простагландин Е1;

б) антациды;

в) прокинетики;

г) гепатопротекторы.

007. НПВС противопоказаны в III триместре беременности так как:

а) Пролонгируют беременность, замедляют родовую деятельность.

б) Вызывают гипотрофию плода.

в) Способствуют нарушению лактации.

008. Пациентам, чьи профессии связаны с повышенным вниманием, не рекомендуется назначать:

а) Индаметацин.

б) Аспирин.

в) Нимесулид.

г) Ибупрофен.

009. В неонатологии для закрытия открытого артериального протока используется:

а) индаметацин.

б) нимесулид.

в) аспирин.

г) фенилбутазон.

010. Синдром Рея возникает при назначении:

а) метамизола.

б) ибупрофена.

в) ацетилсалициловой кислоты.

г) индаметацина.

011. Комбинированное использование НПВС (укажите неверный ответ):

а) возможно для усиления анальгетического и/или антипиретического действия;

б) в ряде случаев ведет к ослаблению эффектов каждого препарата;

в) повышает риск нежелательных реакций;

г) назначается в случае неэффективности монотерапии;

012. Укажите НПВС, с наибольшим влиянием на почечный кровоток и риском нефротоксичности по ишемическому типу:

а) ибупрофен;

б) парацетамол;

в) целекоксиб;

г) диклофенак;

013. НПВС проявляют иммуносупрессивный эффект за счет влияния на:

а) Т-клеточное звено;

б) В-клеточное звено;

в) фагоцитарную активность;

г) капиллярную проницаемость и нарушением контакта антигена с иммунокомпетентными клетками.

014. Наибольшим риском гематотоксичности обладают НПВС из группы:

а) салицилаты;

б) производные индолуксусной кислоты;

в) пиразолидины;

г) производные пропионовой кислоты.

015. При остром отравлении ацетилсалициловой кислотой пациенту назначают:

а) Раствор бикарбоната натрия.

б) 5% раствор глюкозы.

в) Реополиглюкин.

г) Переливание плазмы.

016. Наибольшей минералокортикоидной активностью обладает:

а) гидрокортизон;

б) метилпреднизолон;

в) триамцинолона ацетонид;.

г) дексаметазон.

017. Применение системных глюкокортикостероидов приводит к:

а) лейкопении, эритропении, тромбоцитозу;

б) лейкопении, эритроцитозу, тромбоцитозу;

в) лейкоцитозу, эритропении, тромбоцитопении;

г) лейкопении, эритропении, тромбоцитопении.

018. Влияние глюкокортикоидов на обмен кальция:

а) улучшают всасывание кальция в кишечнике;

б) вызывают гипокальциемию и гиперкальциурию;

в) способствуют накоплению кальция в костях;

г) снижают почечную экскрецию кальция.

019. Системные глюкокортикостероиды при проведении фармакодинамической терапии назначаются:

а) 2/3 дозы в 8.00 и 1/3 в 12.00

б) 1/3 дозы в 8.00 и 2/3 в 12.00

в) 2/3 дозы в 8.00 и 1/3 в 20.00

г) 3 раза в день в равных дозах

020. Укажите системный глюкокортикостероид длительного действия:

а) преднизон;

б) бетаметазон;

в) беклометазона дипропионат;

г) метилпреднизолон.

021. Выберите правильный ответ. Глюкокортикостероиды:

а) практически не влияют на углеводный обмен;

б) усиливают действие вводимого инсулина;

в) усиливают эффект пероральных гипогликемических средств;

г) являются контринсулярными гормонами.

022. Обязательным условием назначения альтернирующей терапии системными глюкокортикостероидами является:

а) отсутствие эффекта от ежедневного назначения глюкокортикостероидов;

б) неэффективность пульс-терапии;

в) стабилизация общего состояния больного;

г) декомпенсированное состояние больного.

023. При проведении заместительной терапии более предпочтителен:

а) преднизолон;

б) метилпреднизолон;

в) дексаметазон;

г) гидрокортизон

024. При лечении бактериального менингита, отека мозга применяется:

а) преднизолон;

б) метилпреднизолон;

в) дексаметазон;

г) гидрокортизон.

025. При 3-х недельном курсе терапии системными стероидами препарат отменяют:

а) Постепенно, по 2.5-5 мг по преднизолону каждые 3-5 дней.

б) Резкой отменой.

в) Постепенно, по 2.5 – 5 мг по преднизолону каждые 2 дня

г) Постепенно, по 10 мг по преднизолону каждые 10 дней.

026. Системные глюкокортикостероиды противопоказаны при:

а) Эпилепсии.

б) Менингите.

г) Тромбоцитопенической пурпуре.

001. Базисная терапия первой линии легкого персистирующего течения бронхиальной астмы включает:

а) Низкие дозы ингаляционных глюкортикостероидов.

б) Кромоны.

в) Антилейкотриеновые препараты

г) Теофиллины длительного действия

002. При обострении бронхиальной астмы препаратом первого выбора на любом этапе оказания медицинской помощи является:

а) эуфиллин.

б) преднизолон.

в) адреналин.

г) сальбутамол.

003. При обострении бронхиальной астмы предпочтительно назначение препаратов:

а) Через небулайзер.

б) Через спейсор.

в) Перорально.

г) Внутривенно.

004. Выберите ингаляционный глюкокортикостероид с наименьшей биодоступностью:

а) флютиказона пропионат;

б) флунизолид;

в) будесонид;

г) триамцинолона ацетонид.

005. При обострении бронхиальной астмы средней степени тяжести обязательно назначение:

а) аминофилина;

б) системного глюкокортикостероида;

в) кромогликата натрия;

г) внутривенное введение?2 –агонистов короткого действия.

а) Антигистаминные препараты.

б) Ингаляционные глюкокортикостероиды.

в) Симпатомиметики.

г) Антихолинергические средства.

007. Снижение объема противовоспалительной терапии возможно при достижении и поддержании контроля бронхиальной астмы в течении:

а) 1 месяца.

б) 3 месяцев.

в) 6 месяцев.

008. Выраженный местный эффект ингаляционных глюкокортикостероидов определяется:

а) высокой липофильностью препаратов;

б) ингаляционным введением препаратов;

в) коротким периодом полувыведения;

г) отсутствием минералокортикоидной активности.

009. Системные глюкокортикостероиды назначаются пациенту с бронхиальной астмой:

а) в качестве альтернативной терапии среднетяжелого течения;

б) как усиление терапии тяжелого течения при неэффективности высоких доз ингаляционных глюкокортикостероидов в комбинации с бета2 –агонистами длительного действия, антилейкотриеновыми препаратами, теофиллинами длительного действия;

в) как средства для купирования приступов одышки;

г) в монотерапии тяжелого течения.

010. Риск развития системных побочных эффектов при применении ингаляционных гдюкокортикостероидов возрастает при назначении доз:

а) выше 2000 мкг/сут;

б) выше 500 мкг/сут;

в) выше 800 мкг/сут;

г) выше 1000 мкг/сут.

011. К селективным бета2 –агонистам длительного действия не относится:

а) формотерол;

б) фенотерол;

в) сальметерол;

г) кленбутирол.

012. Выберите селективный ингаляционный М-холинолитик, применяющийся в лечении бронхообструктивного синдрома:

а) тербуталин;

б) тиотропиум бромид;

в) орципреналин;

г) азаметония бромид.

013. Выберите фармакологическую группу, наиболее предпочтительную для лечения аспириновой астмы:

a) кромоны;

б) системные глюкокортикостероиды;

в) антилейкотриеновые препараты;

г) ингаляционный М-холинолитики.

014. Оптимальный бронходилатирующий эффект теофиллина развивается при поддержании его сывороточной концентрации в диапазоне:

а) 5-10 мкг/дл;

б) 10-15 мкг/дл;

в) 20-35 мкг/дл;

г) 35-60 мкг/дл.

015. Клинический эффект при назначении ингаляционных кортикостероидов больным с астмой обычно отмечается через:

а) 1-2 часа;

в) 4-6 недель;

г) 4-6 месяцев.

016. Применение какого препарата определяет наименьший риск побочных эффектов со стороны сердечно-сосудистой системы:

а) фенотерол;

б) тербуталин;

в) сальбутамол.

017. Ингибитором фермента 5-липооксигеназы является препарат:

а) монтелукаст;

б) зилеутон;

в) сальметерол;

г) недокромил натрия.

018. Выберите бета2-агонист длительного действия:

а) кленбутирол

б) карведилол

в) сальбутамол

г) флютиказона пропионат

019. Серетид - это комбинированный препарат, в состав которого входят:

а) фенотерол + кромогликат натрия;

б) ипратропиум бромид + флютиказона пропионат;

в) сальметерол + флютиказона пропионат;

г) ипратропиум бромид + фенотерол.

020. Назначение бета2 –агонистов в лечении бронхиальной астмы противопоказано при:

а) тиреотоксикозе;

б) эпилепсии;

в) беременности;

г) облитерирующем эндоартериите.

021. Бронхолитический эффект метилксантинов связан с ингибированием фермента:

а) фосфодиэстераза

б) фосфолипаза А2

в) липооксигеназа

г) глутаматсинтетаза

022. Для профилактики развития постнагрузочного бронхоспазма используются все перечисленные препараты за исключением:

а) недокромил натрия;

б) сальметерол;

в) монтелукаст;

г) будесонид.

023. Высокий риск побочных эффектов при применении метилксантинов связан с:

а) узким терапевтическим интервалом

б) невозможностью быстрого выведения препарата форсированным диурезом

в) многочисленными факторами, влияющими на метаболизм

г) верно А и B

д) верно А и C

024. К наиболее значимым фармакодинамическим эффектам ингаляционных кортикостероидов относится все, кроме:

а) торможение синтеза и высвобождения медиаторов аллергического воспаления;

б) потенциирование эффектов эндогенных катехоламинов;

в) восстановление чувствительности адренорецепторов;

г) прямое бронходилатирующее действие.

025. Определите тип фармакодинамического взаимодействия ингаляционных глюкокортикостероидов и бета2 –агонистов длительного действия в лечении бронхиальной астмы:

а) синергизм

б) антагонизм

в) аддитивный эффект

г) не взаимодействуют

026. Пациенту с мерцательной аритмией и бронхиальной астмой для купирования приступов затрудненного дыхания рекомендуется:

а) антилейкотриеновые препараты;

б) ингаляционные М-холинолитики;

в) В2 –агонисты короткого действия;

г) метилксантины.

001. Антитромбоцитарные средства:

б) Тормозят биологическую активность основных факторов свёртывания крови.

в) Блокируют синтез факторов свёртывания крови в печени.

002. Антиагреганты - модуляторы системы аденилатциклаза/цАМФ:

а) Дипиридамол.

б) Тиклопидин, клопидогрель.

в) Ацетилсалициловая кислота.

г) Абциксимаб, эптифибатид, тирофибан.

003. Показанием к назначению аспирина как антиагреганта является:

а) Профилактика тромбозов у больных с ревматическим митральным стенозом.

б) Профилактика тромбозов у больных с язвенной болезнью желудка.

в) Профилактика тромбозов у больных циррозом печени.

г) Профилактика тромбозов у беременных.

004. Какой тип ЦОГ необходимо заблокировать для развития антиагрегантного эффекта?

005. Антиагреганты – блокаторы гликопротеиновых рецепторов для фибриногена GP Iib/IIIa:

а) Абциксимаб, эптифибатид, тирофибан.

б) Дипиридамол.

в) Тиклопидин, клопидогрель.

г) Ацетилсалициловая кислота.

006. Используемые в настоящее время лекарственные формы препаратов из группы блокаторов гликопротеиновых рецепторов GP IIb/IIIa тромбоцитов:

а) Только внутривенные.

б) Только таблетированные.

в) И внутривенные, и таблетированные.

007. Аспирин блокирует циклооксигеназу тромбоцитов:

а) Обратимо.

б) Необратимо.

008. Нагрузочная антиагрегантная доза аспирина составляет:

а) 75-160 мг/сут.

б) 500-1000 мг/сут.

в) 160-325 мг/сут.

009. Противопоказанием к назначению аспирина как антиагреганта является:

а) Профилактика тромбозов у больных с клиническими проявлениями атеросклероза.

б) Язвенная болезнь желудка и двенадцатиперстной кишки.

в) Первичная профилактика ИБС у больных из группы высокого риска.

г) Застойная сердечная недостаточность.

010. К антитромбин-III независимым ингибиторам тромбина относятся:

а) Гирудин.

б) Нефракционированный гепарин,

в) низкомолекулярные гепарины,

г) сулодексид.

011. Механизм действия непрямых антикоагулянтов:

а) Блокируют фермент витамин К-редуктазу.

б) Образуют комплекс с антитромбином III.

в) Блокируют фермент ЦОГ-1.

г) Блокируют фермент витамин К-синтетазу.

012. Нефракционированный гепарин инактивирует следующие факторы свёртывания:

а) IIa, IXa, Xa, XIa, XIIa.

б) IIa, IXa, Xa.

013. Антикоагулянтный эффект нефракционированный гепарин реализует:

а) Непосредственно действуя на факторы свёртывания.

б) Образуя комплекс с тканевым активирующим фактором.

в) Образуя комплекс с антитромбином III.

г) Образуя комплекс с протромбином.

014. Антикоагулянты прямого действия:

а) Тормозят адгезию и агрегацию тромбоцитов.

б) Блокируют синтез факторов свёртывания крови в печени.

г) Растворяют фибриновый тромб.

015. Антикоагулянты непрямого действия:

а) Блокируют синтез факторов свёртывания крови в печени.

в) Тормозят биологическую активность основных факторов свёртывания крови.

г) Растворяют фибриновый тромб.

016. Наиболее чувствительны к инактивации комплексом гепарин/антитромбин III факторы:

017. В результате действия непрямых антикоагулянтов нарушается печёночный синтез следующих факторов свёртывания:

а) II, VII, IX, X.

б) II, VIII, IX, XI.

в) VII, IX, XI, XII.

г) II, VII, XI, XII.

018. К антикоагулянтам прямого действия относятся:

а) Нефракционированный гепарин, низкомолекулярные гепарины, сулодексид.

б) Ингибиторы циклооксигеназы; тиенопиридины; модуляторы системы аденилатциклаза/цАМФ; блокаторы гликопротеиновых рецепторов для фибриногена GP IIb/IIIa.

в) Монокумарины, дикумарины, индандиноны.

г) Активаторы эндогенного плазминогена.

019. Противопоказанием к назначению антикоагулянтов непрямого действия с целью профилактики и лечения тромбоэмболических осложнений является:

а) Язвенная болезнь желудка и двенадцатиперстной кишки в стадии обострения.

б) Мерцательная аритмия.

в) Протезированные клапаны сердца.

г) Тромб в полости левого желудочка.

020. Эффективным методом борьбы с развившимся кровотечением на фоне терапии непрямыми антикоагулянтами является:

а) Свежезамороженная плазма.

б) Викасол.

в) Протамина сульфат.

г) Аминокапроновая кислота.

021. Контролируемый параметр терапии низкомолекулярными гепаринами:

а) Активированное частичное тромбопластиновое время (АЧТВ).

б) Время свёртывания крови.

в) Международное нормализационное отношение (МНО).

022. Контролируемый параметр терапии антикоагулянтами непрямого действия:

а) Международное нормализационное отношение (МНО).

б) Активированное частичное тромбопластиновое время (АЧТВ).

в) Время свёртывания крови.

г) Протромбиновый индекс (ПТИ).

023. Противопоказанием к назначению низкомолекулярного гепарина является:

а) Диабетическая геморрагическая ретинопатия.

б) Подготовка к проведению кардиоверсии у пациентов с мерцательной аритмией.

в) Профилактика и лечение тромбозов глубоких вен нижних конечностей.

г) Профилактика и лечение тромбозов у больных с протезированными клапанами сердца.

024. Фибринолитические средства:

а) Растворяют фибриновый тромб.

б) Тормозят адгезию и агрегацию тромбоцитов.

в) Тормозят биологическую активность основных факторов свёртывания крови.

г) Блокируют синтез факторов свёртывания крови в печени.

025. Фибринолитики противопоказаны в случае:

а) Тромбоза глубоких вен нижних конечностей

б) Острого коронарного синдрома с подъёмом сегмента ST.

г) Тромбоза центральной артерии и вены сетчатки.

001. Выберите наиболее рациональную тактику назначения антацидов:

а) чередование приема всасывающихся и невсасывающихся;

б) использование только всасывающихся антацидов;

в) использование только невсасывающихся антацидов;

г) использование невсасывающихся антацидов при неэффективности всасывающихся антацидов.

002. Наиболее оптимален прием антацидов:

а) перед едой;

б) во время еды;

в) между приемами пищи;

г) вне зависимости от приема пищи.

003. При длительном приеме высоких доз невсасывающихся антацидов возможно появление нежелательных реакций (исключите неверный ответ):

а) повышение риска нефролитиаза;

б) гипергастринемия;

в) гипофосфатемия;

г) деменция при нарушении функции почек.

004. Комбинация гидроокиси магния и гидроокиси алюминия обеспечивает:

а) усиление антацидного действия;

б) усиление послабляющего эффекта;

в) быстрое начало антацидного действия в сочетании с его достаточной продолжительностью;

г) повышение всасывания компонентов препарата.

005. Основной механизм действия ингибиторов протонной помпы:

а) адсорбция соляной кислоты;

б) нарушение образования соляной кислоты;

в) блокада рецепторов, регулирующих секрецию соляной кислоты;

г) нейтрализация соляной кислоты химическим путем.

006. Протонная помпа – это фермент:

а) Н+К+-АТФаза

б) гуанилатциклаза

в) Na+К+-АТФаза

г) фосфодиэстераза

007. Ингибиторы протонной помпы:

а) увеличивают продукцию гастрина;

б) тормозят продукцию гастрина;

в) в низких дозах тормозят, в высоких – увеличивают;

г) не влияют на продукцию гастрина.

008. Выберите препарат из группы ингибиторов протонного насоса:

а) мизопростол

б) пирензепин

в) фамотидин

г) рабепразол

009. Наиболее активный ингибитор цитохрома Р-450:

а) циметидин

б) ранитидин

в) фамотидин

г) роксaтидин

010. Укажите нежелательный эффект, развивающийся на фоне приема фамотидина:

а) гематотоксичность

б) антиандрогенное действие

в) гепатотоксичность

г) синдром рикошета

011. Какое влияние на эндокринную систему оказывают Н2-гистаминоблокаторы 1 поколения?

а) антиандрогенное действие

б) антиэстрогенный эффект

в) антигонадотропный эффект

г) торможение выработки вазопрессина

012. Выберите препарат из группы селективных М1,2-холинолитиков:

а) мизопростол

б) платифиллин

в) цезаприд

г) пирензепин

013. Наиболее выраженной антисекреторной активностью обладают:

а) ингибиторы протонной помпы

б) Н2-гистаминоблокаторы

в) селективные М-холинолитики

г) синтетические простагландины

014. Пилорид – это комбинированный препарат, включающий:

а) ранитидина висмута субцитрат

б) фамотидина висмута субцитрат

в) препараты трехкомпонентной схемы эрадикационной терапии в рекомендованных дозах

г) омепразола висмута субцитрат

015. Основное показание к назначению мизопростола:

а) лечение дуодено-гастрального рефлюкса

б) лечение язвенной болезни желудка

в) совместное применение с НПВС для снижения гастротоксичности

г) лечение функциональной диспепсии

016. Основными фармакодинамическими эффектами мизопростола являются (исключить неверный ответ):

а) уменьшение синтеза соляной кислоты

б) повышение регенерации

в) увеличение выработки простагландинов

г) бактерицидное действие на H. pylori

017. В трехкомпонентной схеме эрадикационной терапии H.pylori у детей старше 14 лет рекомендуется назначить:

а) ингибитор протонной помпы

б) Н2-гистаминоблокатор

в) селективный М-холинолитик

г) синтетический простагландин Е2

а) Амоксициллин + Кларитромицин + Коллоидный субцитрат висмута;

б) Амоксициллин + Кларитромицин + Омепразол;

в) Амоксициллин + Рокситромицин + Метронидазол;

г) Амоксициллин +Кларитромицин + Ранитидин.

019. Абсолютным показанем к проведению эрадикационной терапии является:

а) гипертрофический гастрит;

в) дуоденогастральный рефлюкс;

а) 5 - 7 дней

б) 7 - 10 дней

в) 10 - 14 дней

021. Контроль эрадикации проводится через:

а) 10 дней после терапии

б) 2 недели после терапии

в) 4 недели после терапии

г) 6-8 недель после терапии

022. Укажите препарат не активный в отношении H. pylori:

а) метронидазол.

б) эритромицин.

в) кларитромицин.

г) тетрациклин.

023. Укажите показание к бактериологическому исследованию биоптата слизистой желудка и определения чувствительности H. Pylori к антибиотикам:

а) язвенная болезнь желудка и 12-перстной кишки;

б) рак желудка;

в) длительная терапия НПВС или ГКС;

024. Укажите верное утверждение:

а) домперидон повышает напряжение нижнего пищеводного сфинктера, сократимость антральной части желудка и повышает тонус пилорического сфинктера;

б) домперидон плохо проникает через ГЭБ, что не снижает его терапевтический эффект, но увеличивает количество побочных эффектов;

в) домперидон чаще по сравнению с метоклопрамидом вызывает экстрапирамидные нарушения;

г) домперидон снижает напряжение нижнего пищеводного сфинктера, сократимость антральной части желудка и снижает тонус пилорического сфинктера.

РАЗДЕЛ VIII. КЛИНИЧЕСКАЯ ФАРМАКОЛОГИЯ АНТИМИКРОБНЫХ ПРЕПАРАТОВ И ИХ ПРИМЕНЕНИЕ В ПЕДИАТРИИ.

001. Спектр активности аминогликозидов включает преимущественно:

а) Грамотрицательную аэробную микрофлору.

б) Грамположительную аэробную микрофлору

в) Грамположительную анаэробную флору.

г) Атипичных внутриклеточных возбудителей.

002. Укажите препарат из группы цефалоспоринов эффективный в лечении менингита, вызванного менингококком:

а) цефтриаксон.

б) цефазолин.

в) цефуроксим.

г) цефалексин.

003. Наиболее высокая биодоступность при пероральном приеме характерна для:

а) Амоксициллина.

б) Оксациллина.

в) Ампициллина.

г) Феноксиметилпенициллина.

004. При избыточной массе тела (более 20%) суточная доза аминогликозидов:

а) рассчитывается на фактический вес.

б) рассчитывается на идеальный вес.

в) аминогликозиды противопоказаны к назначению.

005. Выберите препарат активный в отношении энтерококковой инфекции:

а) Амоксициллин.

б) Цефазолин.

в) Азитромицин.

г) Цефипим.

006. Выберите препарат для лечения микоплазменной инфекции у ребенка 6 лет:

а) Доксициклин.

б) Моксифлоксацин.

в) Кларитромицин.

г) Амикацин.

007. Эффективность антибактериальной терапии оценивается:

а) Через 2-3-е суток после назначения препарата.

б) В первые сутки назначения препарата.

в) Через 5-6 суток после назначения препарата.

008. Укажите макролид, на биодоступность которого значимо влияет присутствие пищи:

а) Эритромицин.

б) Спирамицин.

в) Джозамицин.

г) Кларитромицин.

009. Развитие «серого» синдрома характерно при назначении новорожденным:

а) Хлорамфеникола.

б) Гентамицина.

в) Цефтриаксона.

г) Тетрациклина.

010. Укажите препарат выбора для эмпирической терапии внебольничной пневмонии у детей с 1 по 6 месяц жизни, при низкой вероятности атипичных возбудителей:

а) Амоксициллина/клавуланат.

б) Амоксициллин.

в) Гентамицин.

г) Левофлоксацин

д) Бензилпенициллин.

011. Механизм действия макролидов заключается:

а) в специфическом ингибировании синтеза клеточной мембраны

б) в подавлении синтеза белка на уровне рибосом

в) в ингибировании синтеза РНК

г) в нарушении метаболизма фолиевой кислоты

012. Применение клавулановой кислоты в сочетании с амоксициллином позволяет:

а) расширить спектр действия амоксициллина

б) снизить токсичность амоксициллина

в) сократить частоту приема амоксициллина

г) увеличить проникновение амоксициллина через ГЭБ

013. Из перечисленных антибиотиков укажите препарат выбора для лечения инфекции, вызванной метициллинрезистентным стафилококком:

а) ципрофлоксацин

б) имипенем

в) ванкомицин

г) цефипим

014. Какой из перечисленных антибиотиков обладает наименьшим нефротоксическим действием:

а) гентамицин

б) ванкомицин

в) эритромицин

г) стрептомицин

015. Выберите препарат из группы цефалоспоринов, имеющий форму для перорального приема:

а) цефепим;

б) цефиксим;

в) цефазолин;

г) цефотаксим.

016. Препараты из группы цефалоспоринов III поколения отличаются от цефалоспоринов I поколения:

а) расширением спектра активности в отношении грамотрицательной флоры;

б) большей токсичностью препаратов;

в) расширением спектра активности в отношении грамоположительной флоры;

г) меньшей устойчивостью к продукции микроорганизмами бета-лактамаз.

017. Механизм действия фторхинолонов заключается:

а) в специфическом ингибировании синтеза клеточной мембраны;

б) в подавлении синтеза белка на уровне рибосом;

в) в ингибировании ДНК-гиразы и нарушении репликации ДНК;

г) в нарушении метаболизма фолиевой кислоты.

018. Наиболее рациональным в эмпирической терапии внебольничной пневмонии у детей старше 1 года является комбинация аминопенициллинов с:

а) цефалоспоринами III поколения;

б) ингибиторами бета-лактамаз;

в) макролидами;

г) аминогликозидами.

019. Препараты из группы макролидов не обладают активностью в отношении:

а) стрептококков;

б) возбудителей коклюша и дифтерии;

в) золотистого стафилококка;

г) синегнойной палочки.

020. Наиболее характерным побочным эффектом пенициллинов является:

а) нефротоксичность;

б) аллергические реакции;

в) гепатотоксичность;

г) тератогенность.

021. Комбинация гентамицина с ванкомицином считается:

а) рациональной, вследствие повышения антибактериальной активности обоих препаратов;

б) нерациональной из-за риска высокой гепатотоксичности;

в) нерациональной из-за снижения антибактериальной активности;

г) нерациональной из-за риска высокой нейро- и нефротоксичности.

022. Препараты из группы макролидов оказывают:

а) бактерицидное действие;

б) бактериостатическое действие;

в) преимущественно бактериостатическое действие, но в отношении некоторых штаммов действуют бактерицидно;

г) преимущественно бактерицидное действие, но в отношении некоторых штаммов действуют бактериостатически.

023. К группе цефалоспоринов III поколения с выраженной антисинегнойной активностью относится:

а) цефтазидим

б) цефиксим

в) цефтриаксон

г) цефотаксим.

024. Активностью против атипичных возбудителей (микоплазм, хламидий, легионелл) не обладает:

а) ципрофлоксацин

б) доксициклин

в) ампициллин

г) кларитромицин.

025. Укажите антибактериальный препарат, обладающий наибольшей активностью в отношении анаэробной флоры:

а) гентамицин

б) тетрациклин

в) ампициллин

г) метронидазол.


ЭТАЛПНЫ ОТВЕТОВ

РАЗДЕЛ I. ОБЩИЕ ВОПРОСЫ КЛИНИЧЕСКОЙ ФАРМАКОЛОГИИ.

РАЗДЕЛ II. КЛИНИЧЕСКАЯ ФАРМАКОЛОГИЯ СРЕДСТВ, ВЛИЯЮЩИХ НА СОСУДИСТЫЙ ТОНУС. ЛЕЧЕНИЕ АРТЕРИАЛЬНОЙ ГИПЕРТЕНЗИИ У ДЕТЕЙ.

РАЗДЕЛ III. КЛИНИЧЕСКАЯ ФАРМАКОЛОГИЯ АНТИАРИТМИЧЕСКИХ ПРЕПАРАТОВ И ИХ ПРИМЕНЕНИЕ В ПЕДИАТРИИ.

РАЗДЕЛ IV. КЛИНИЧЕСКАЯ ФАРМАКОЛОГИЯ НПВС И ГКС.

ЗАНЯТИЕ V. КЛИНИЧЕСКАЯ ФАРМАКОЛОГИЯ СРЕДСТВ, ПРИМЕНЯЕМЫХ ПРИ СИНДРОМЕ БРОНХООБСТРУКЦИИ У ДЕТЕЙ.

РАЗДЕЛ VI. СРЕДСТВА, ВЛИЯЮЩИЕ НА ГЕМОСТАЗ И РЕОЛОГИЮ.

РАЗДЕЛ VII. КЛИНИЧЕСКАЯ ФАРМАКОЛОГИЯ СРЕДСТВ, ВЛИЯЮЩИХ НА МОТОРИКУ И СЕКРЕЦИЮ ЖКТ.

(=#)РАЗДЕЛ VIII. КЛИНИЧЕСКАЯ ФАРМАКОЛОГИЯ АНТИМИКРОБНЫХ ПРЕПАРАТОВ И ИХ ПРИМЕНЕНИЕ В ПЕДИАТРИИ.

Выберите правильное утверждение: а)биодоступность-количество ЛС,поступающее в системный кровоток,выраженное в процентах от введенной дозы,б)биодоступность определяется величиной адсорбции ЛС в ЖКТ и выраженностью эффекта первого прохождения через печень.в)биодоступность определяют по формуле: F = AUC (в/м или внутрь)/AUC (в/в).г)биодоступность ЛС при внутримышечном введении определяется степенью его всасывания и биотрансформации в организме.
Ответ: а, б, в

2.
Ответ: Атровент

3.

Ответ: а, д

4.

Ответ:

5.
Ответ:

6.

д) Ксилит
Ответ: а,в

7.

Ответ: а,б,д

8.
Ответ:

9. поступил с острым инфарктом миокарда,возникшим 5ч назад.Назначения: анаприлин 20 мг 4 раза в сутки внутрь, гепарин в/в капельно по 10 000 ЕД каждые 4 часа.При этом удалось достигнуть увеличения времени свертывания крови до 18-23 минуты. На следующий день у б-ного диагностирована правосторонняя нижнедолевая пневмония.Назначена натриевая соль бензилпенициллина (по 1000 000 ЕД каждые 4 ч) в/в.Через 4 ч время свертывания крови составило 8 минут. Какова ваша тактика?
Ответ:

10.

11.
Ответ: Вит.В12 в дозе 500 мкг/сут через день, фолиевая кислота в дозе 1,5 мг/сут, сульфат железа(80 мг Fe2+) 1 раз в сутки

12.

Ответ: Вит.С

13.

Ответ: Церебролизин

14.
аллергию (на бутадион, гепарин, метиндол, пенициллин, теофиллин).В стаци-онаре б-ной назначены реопирин по 5 мл в/м 1 раз в сутки,гидрокортизона гемисукцинат по 100 мг в полость коленных суставов,тавегил по 0,001 г 2р.в сутки.Через 3 дня у б-ной появились зудящие эритематозные высыпания на коже туловища.Какова наиболее вероятная причина
ухудшения состояния?
Ответ:

15.



Ответ: а,б,д,е,з,и

16.
Ответ: Через несколько месяцев

17.


Ответ: а,б,в,д,е

18.
Ответ: а,б,в,г,д,ж,з

19.
Ответ:

20.
Ответ: Ципрофлоксацин

21. Феномен первого прохождения ЛС через печень зависит от:а) кровоснабжения печени, б) связывания ЛС с белком, в) активности ферментов гепатоцитов, г) уровня экскреции ЛС,д) быстроты всасывания
Ответ: а, в

22. Препараты, влияющие на микросомальные ферменты печени:индукторы микросомальных ферментов печени: а) пенициллин, б) нитроглицерин, в) фенобарбитал, г) фуросемид, д) бутадион, е) кортизол, ж) пропранолол, з) циметидин, и) левомицетин, к) дифенин
Ответ: в,д

23. поступила в отделение с ж-ми на боли в правой молочной железе, повышение Т.до 39,5 С.Заболела 3 дня назад,на 10-й день после родов. При поступлении в отделение в верхненаружном квадранте правой молочной железы обнаружена гиперемия кожи,массивный инфильтрат с флуктуацией в центре.Диагноз:острый правосторонний мастит.Б-ная оперирована. Взят посев отделяемого раны.Определите антибиотик первого выбора
Ответ: Цефазолин

24.

Ответ: Анафилактическая реакция

25.

Ответ: Левомицетин

26.
Ответ:

27.
Ответ: Бигуаниды

28.

Ответ: Гипотония, головокружение.

29.

Оцените действия врача.

30. У больного Д., 53 лет, диагноз: ИБС, стабильная стенокардия??? ФК, постинфарктный кардиосклероз, мерцательная аритмия,ХНК??Б ст. Принимал строфантин, дигоксин, фуросемид, панангин в средних терапевтических дозах. Неожиданно у больного повысилась температура до 38,4°С, появилась кашель, одышка, крепитация в легких справа. На рентгенограмме легких справа в нижней доле определяется участок инфильтрации. К лечению добавлены гентамицин, сульфокамфокаин,супрастин.

Ответ:

31.

Ответ: Фентоламин.

32.

Ответ: г,д

33.

Ответ:

34.
Ответ:

35.
Ответ: Эналаприл.

36.

Ответ: а,б,г

37.

Ответ:

38.
Ответ:

39.
Ответ:

40.

Ответ: все перечисленное

41. Препараты, влияющие на микросомальные ферменты печени: ингибиторы микросомальных ферментов печени:
а) пенициллин, б) нитроглицерин,в) фенобарбитал,
г) фуросемид, д) бутадион, е) кортизол, ж)пропранолол,
з) циметидин, и) левомицетин, к) дифенин
Ответ: з,и

42.
Ответ: Через 7-14 дней

43. Укажите комбинацию препаратов, приводящую к возникновению конкуренции за связывание с белком, что может обусловить опасное повышение содержания свободной фракции одного из препаратов в крови и появление симптомов его передозировки:
Ответ: неодикумарин и бутадион

44. Выберите ЛС с узким терапевтическим диапазоном:
а) пенициллины, б) противосудорожные средства,
в) антиаритмические средства,г) дигоксин, д) метотрексат, е) теофиллин, ж) циклоспорин, з) макролиды
Ответ: б,в,г,д,е,ж

45. Укажите комбинации препаратов, при которых вследствие конкуренции за связь с белком происходит увеличение концентрации в плазме крови свободной фракции одного из них: а.строфантин и мисклерон, б.дигитоксин и мисклерон, в.неодикумарин и бутадион, г.нифедипин и гидрохлортиазид
Ответ: б,в

46. выявлены частая желудочковая экстрасистолия и пароксизмы мерцательной аритмии.ЧСС 74 в мин,АД 140/80 мм.рт.ст.Последние 3г.беспокоят приступы стенокардии напряжения и покоя.Лечение проводили кордароном.Учитывая побочные эффекты назначенного препарата,выберите ЛС для дальнейшего лечения б-ного: а) Хинидин,
б) Боннекор, в) Этацизин,

Ответ: а,б

47. Известно,что при комбинации хинидина и дигоксина часто наблюдается гликозидная интоксикация.С чем она связана? Фармакодинамическое взаимодействие:
Ответ: синергизм

48. Известно,что при комбинации хинидина и дигоксина наблюдается гликозидная интоксикация.С чем она связана? Фармакокинетическое взаимодействие, влияние хинидина на:
Ответ: связь с белком

49. Критические периоды внутриутробного развития:
а. период предимплантационного развития (1 нед)
б. стадия эмбриогенеза заканчивается к 8 нед.
в. стадия эмбриогенеза заканчивается к 8 мес.
г. период непосредственно перед родами
Ответ: а,б,г

50. Выберите из нижеуказанных препаратов,обладающие перечисленными свойствами: Противомикробные препараты,применение которых практически безопасно при беременности: а.сульфаниламды,в том числе бисептол,
б.аминогликозиды,тетрациклины,рифампицины,метронидазол (в 1триместр беременности), в. пенициллины, цефалоспорины, эритромицин,линкомицин,фузидин, г.антимикотические средства,противоопухолевые
антибиотики.
Ответ: в

51. Метронидазол назначен кормящей матери, укажите побочные эффекты:
а.повышение возбудимости,тахикардия, б.угнетение аппетита,рвота, в.угнетение ЦНС,дыхания,снижение массы тела, г.увеличение секреции пролактина,нагрубание молочных желез, д.гипоплазия надпочечников, нарушение обмена веществ,повышение риска развития билирубиновой энцефалопатии, е.геморрагии,нарушение дыхания,ацидоз, ж.угнетение кроветворения,анемия, гипотрофия, дисбактериоз.
Ответ: б

52. Антимикробные препараты первого выбора у новорожденных: а.бензилпенициллин,оксациллин,карбенициллин,гентамицин,амикацин,б.бензилпенициллин,оксациллин,бициллины,цефазолин,цефотаксим,эритромицин,линкомицин,нистатин, в.карбенициллин,гентамицин,сизомицин,амикацин, тобрамицин,цепорин (при неэффективности цефалоспоринов первого поколения), г.эритромицин,линкомицин,нистатин,леворин,карбенициллин,
гентамицин, сизомицин
Ответ: б

53.
Ответ:

54. Основные особенности фармакокинетики ЛС у лиц пожилого возраста:
а.снижение скорости абсорбции, б.ускорение абсорбции, в.снижение скорости распределения, г.ускорение распределения, д.уменьшение связывания ЛС с белками плазмы, е.увеличение связывания ЛС с белками плазмы, ж.замедление метаболизма, з.ускорение метаболизма,
и.замедление выведения ЛС, к. ускорение выведения ЛС.
Ответ: а,в,д,ж,и

55.
Ответ: б,в,г

56. Укажите побочные эффекты бета-адреноблокаторов: а)брадикардия, б)артериальная гипотензия, в)бронхоспазм, г)тахикардия, д)нарушение функций щитовидной железы,
е) перемежающаяся хромота, ж) АВ-блокада
Ответ: а,б,в,е,ж

57.
состояния: а)Естественное течение заболевания, б)Развитие толерантности к нитратам, в)Синдром межкоронарного обкрадывания, г) Возникновение синдрома рикошета д) явления идиосинкразии
Ответ: а,б

58. Укажите побочные эффекты амиодарона: а) брадикардия, б).артериальная гипотензия, в) бронхоспазм,г)тахикардия, д) нарушение функций щитовидной железы, е) перемежающаяся хромота, ж) АВ-блокада
Ответ: а,в,д,ж

59. Как изменится ваша антиангинальная терапия,если у б-ного на фоне терапии нитратами возникнет инсульт головного мозга?
Ответ: отмена нитратов и назначение антиангинального препарата другой группы

60. Какие гипотензивные препараты считают наиболее безопасными для пожилых больных: а)бета-адреноблокаторы, б)ганглиоблокаторы, в)симпатолитики, г)блокаторы медленных кальциевых каналов, д)тиазидные
диуретики, е) ингибиторы АПФ.
Ответ: г,д

61. Схема лечения кордароном:
Ответ: по схеме, предполагающей постепенное снижение дозы с 600 мг до 200 мг в сутки

62. Как влияют ингибиторы МАО (антидепрессанты) на прессорный эффект адреностимуляторов прямого и непрямого действия?
Ответ: усиливают действие

63. Небензодиазепиновый» агонист бензодиазепиновых рецепторов:
Ответ: Золпидем

64. Снотворное средство — соединение алифатического ряда:
Ответ: Хлоралгидрат

65.

Ответ: А (б)

66. Протамина сульфат назначают при передозировке:
Ответ: Гепарина

67. Какой способ детоксикации наиболее эффективен при отравлении веществами, связывающими с белками и липидами крови?
Ответ: Гемосорбция

68. Принцип действия налоксона при остром отравлении морфином:
Ответ: Препятствует действию морфина на опиоидные рецепторы

69. Укажите препараты,обладающие свойствами антиоксидантов: а)верапамил б) вит.А, в) вит.К, г) вит.С, д) вит.Е, е)селен, ж) карнозин, з) доксициклин
Ответ: б,г,д,е,ж

70. Какие эффекты характерны для нейролептиков?
а) антипсихотический, б) седативный, в) противорвотный
Ответ: а,б,в

71. У Б-ой 64л.развился острый приступ закрытоугольной глаукомы с сильными болями в правом глазу с иррадиацией в голову. Появились тошнота и рвота,одышка,выявлены признаки гипертонического криза 2 типа с ЧСС 62
в мин.АД 200/140 мм.рт.ст.Б-ная в течение многих лет страдает гипертонической болезнью.В легких большое кол-во влажных мелкопузырчатых хрипов.Какие диуретики показаны больной? а. Клопамид, б.Верошпирон,в.Гипотиазид, г.Фуросемид в/в, д. Диакарб:
Ответ: г,д

72. в течение 15 лет страдает сахарным диабетом,по поводу которого получает инсулин по 70 ЕД/сут,что поддерживает уровень гликемии в пределах 7,5-8,6ммоль/л.В последнее время стало повышаться АД до
170/90-180/100 мм.рт.ст.в связи с чем лечащий врач назначил обзидан в суточной дозе 120 мг.Какие побочные эффекты следует ожидать при данной комбинации ЛС? а.Гипергликемия вплоть до комы, б.Сердечная недостаточность, в.Гипогликемия вплоть до комы, г.Ортостатическая гипотензия, д. Гипертензия
Ответ: б,в

73. , страдающему гормонально-зависимой бронхиальной астмой,назначили преднизолон (по 5 мг ежедневно),сальбутамол (ингаляция 2 доз аэрозоля 4р.в сутки).В связи с проявлениями судорожного синдрома (в анамнезе черепно-мозговая травма) был назначен фенобарбитал.Через неделю у б-го развилось обострение бронхиальной астмы.С чем это связано?
А. Фенобарбитал ускорил биотрансформацию: а.сальбутамола, б.преднизолона, Б.Фенобарбитал ускорил экскрецию: а.сальбутамола, б.преднизолона, В.Фенобарбитал замедлил экскрецию: а.сальбутамола, б.преднизолона, Г.Фенобарбитал замедлил биотрансформацию:а.сальбутамола,б.преднизолона
Ответ: А (б)

74. страдает ИБС,стенокардией напряжения III ФК.ЧСС 90 в мин,АД 150/80мм.рт.ст.В анамнезе-хронический бронхит с бронхоспастическим синдромом в стадии ремиссии.Жировая дистрофия печени.Укажите группы ЛС (второй этап выбора лекарственной терапии),оптимальные для антиангинальной терапии. а.Нитраты и верапамил, б. Нитраты и атенолол
в) Нитраты и анаприлин, г) Нитраты и нифедипин,
д) Нифедипин и амиодарон
Ответ: а

75. по поводу стенокардии принимает нитросорбид по 10мг 4р.в сутки ЧСС 80 в ми.АД 140/80мм.рт.ст.Через 1 мес.после начала терапии вновь участились приступы стенокардии.Каковы возможные причины ухудшения
состояния: а)Естественное течение заболевания, б)Развитие толерантности к нитратам, в)Синдром межкоронарного обкрадывания, г) Возникновение синдрома рикошета, д) явления идиосинкразии
Ответ: а,б

76. отмечаются приступы стенокардии при умеренных физических нагрузках.В анамнезе коллаптоидное состояние после однократного приема нитроглицерина сублингвально (с тех пор б-ной нитроглицерин не принимал). Сопутствующие заболевания-ГБ (рабочий уровень АД 160/100мм.рт.
ст.,гипофункция щитовидной железы.На момент осмотра АД 190/100мм.рт.ст.,ЧСС 72 в минуту.Больному противопоказаны:
Ответ: Амиодарон

77. по поводу артериальной гипертензии 2степени получает 0,000075г клофелина 4р.в сутки.В связи с развитием сенильной депрессии назначен мелипрамин.Через 3 дня после назначения мелипрамина у б-ного возник гипертонический криз.Накануне б-ной не принимал клофелин.Каковы возможные причины ухудшения состояния: а)Следствие естественного течения заболевания, б)Следствие гипертензивного эффекта мелипрамина, в)Следствие неблагоприятного взаимодействия ЛС, г)Следствие возможного прекращения приема ЛС б-ным и развития синдрома отмены.
Ответ: б,в,г

78. по поводу гипертонического криза вводили натрия нитропруссид в/в капельно в больших дозах (со скоростью 8 мкг/мин). Появились одышка, акроцианоз,давящие боли за грудиной,мышечные подергивания.Какова причина ухудшения состояния б-ного?
Ответ: Токсическое действие цианидов

79. выявлены частая желудочковая экстрасистолия и пароксизмы мерцательной аритмии.ЧСС 74 в мин,АД 140/80 мм.рт.ст.Последние 3г.беспокоят приступы стенокардии напряжения и покоя.Лечение проводили кордароном.Учитывая побочные эффекты назначенного препарата,выберите ЛС
для дальнейшего лечения б-ного: а) Хинидин, б) Боннекор, в) Этацизин,
г) Мекситил, д) Верапамил, е) Пропранолол
Ответ: а,б

80. пароксизмальная суправентрикулярная тахикардия на фоне синдрома WPW.Для купирования приступа выбран аймалин.Определите оптимальную схему лечения выбранным препаратом: а)1 мг/кг в/в в течение более10 мин,при необходимости повторить через 30 мин, б)50 мг в/в в тече-
ние 3-5 мин в 10 мл 5% раствора глюкозы или изотонического раствора NaCl или в/м, в) 0,5-1 г в/в каждые 2 мин вводят по 0,1-0,2 г или в/м
г) после парентерального введения назначить внутрь по 100мг 4-5 раз в сутки, поддерживающая доза 50 мг 3-4 раза в сутки
Ответ: а,г

81. У б-ной 28л.с диагнозом СКВ на фоне ХПН появились отеки голеней,увеличение печени.При эхокардиографическом исследовании определяется снижение сердечного выброса. ЧСС 95 в мин,АД 170/100 мм.рт.ст.Какие сердечные гликозиды показаны больной?
Ответ: Дигитоксин

82. У б-ной 28л.с диагнозом СКВ на фоне ХПН появились отеки голеней,увеличение печени.При эхокардиографическом исследовании определяется снижение сердечного выброса.ЧСС 95 в мин, АД 170/100 мм.рт.ст. Больная принимает дигитоксин.В связи с появлением судорожного синдрома дополнительно назначен фенобарбитал (0,3 г/сут).Когда возникнут изменения в состоянии б-ной при наличии влияния?
Ответ: Через 7-14 дней

83. 57 лет по поводу постинфарктного атерокардиосклероза,застойной сердечной недостаточности 2 степени получает 40 мг фуросемида в/в и 300 мг
верошпирона внутрь.Какую диуретическую терапию Вы назначите б-ному в случае рефрактерности?
Ответ: Фуросемид 80 мг в/в и спиронолактон 300 мг внутрь

84. страдает неатопической бронхиальной астмой,сопровождающейся обильной бронхореей.Пульс 62 в мин.АД 140/80 мм.рт.ст.Какие препараты более предпочтительны?
Ответ: Атровент

85. упорно рецидивирующий синдром бронхиальной обстукции со сниженной чувствительностью к холино и адренотропным средствам.Бронхиальной астмой страдает более 10 лет.Что можно назначить для уменьшения частоты и тяжести приступов бронхиальной астмы: а)Ингаляции бета
2-адреностимуляторов более 6 раз в сутки, б) Ингаляция м-холиноблокатора, в) Введение адреналина п/к в большей дозе,чем обычно,для снятия бронхоспазма, г)Эуфиллин в/в, д) Ингаляционные глюкокортикоиды.
Ответ: г,д

86. поступил с ж-ми на изжогу,боли в эпигастральной области натощак, купируемые приемом натрия гидрокарбоната.При ФЭГДС выявлена язва (0,5см в диаметре) в ампуле 12 п.к.РН-метрия желудочного сока: кислотообра-
зующая функция средней интенсивности с низкими щелочными резервами,холинергический тип рецепции. Диагноз: язвенная болезнь 12 п.к.в стадии обострения. Выберите наиболее эффективное и безопасное ЛС и определите режим его дозирования:
Ответ: Пирензепин до еды по 0,05 г 3 раза в сутки в течение 2дней, затем по 0,05 г 2 раза в сутки

87. выявлена дискенезия желчного пузыря по гипертоническому типу. Выберите оптимальный вариант лечения.
Ответ: Но-шпа по 1-2 табл.3 раза в сутки, отвар бессмертника по 1/2 стакана за 30 мин до еды

88. страдает хрон.холецистопанкреатитом в течение 5 лет.За последнюю неделю после нарушения диеты отмечает усиление болей в правом по дреберье,тошноту,горечь во рту.Выберите наиболее эффективные желчегонные средства, обладающие одновременно противомикробной активностью:
а) Аллохол, б) Холензим, в) Никодин, г) Отвар пижмы,д) Ксилит
Ответ: а,в

89. с суицидальной целью выпила 20 таблеток феназепама.Через 2 ч после приема препарата доставлена в стационар. Б-я в сознании,но резко заторможена.Проведено промывание желудка.Выберите наиболее оптимальные слабительные средства: а) Глауберова соль, б) Сульфат магния, в) Экстракт коры крушины, г) Бисакодил,
д) Касторовое масло, е) Морская капуста, ж) Вазелиновое масло
Ответ: а,б,д

90. 46 лет поступил в отделение кардиореанимации с острым трансмуральным инфарктом миокарда, возникшим около 5 ч назад.Назначения:анаприлин 20 мг 4 раза в сутки внутрь, гепарин в/в капельно по 10 000 ЕД каждые 4 часа.При этом удалось достигнуть увеличения времени свертывания крови до 18-23 минуты. На 4-й день у б-ного выявлена микрогема турия (22 эритроцита в поле зрения). Какова ваша тактика?
Ответ: Снизить дозу гепарина вплоть до времени свертывания крови не менее 10-12 мин

91. поступил с острым инфарктом миокарда,возникшим 5ч назад.Назначения: анаприлин 20 мг 4 раза в сутки внутрь, гепарин в/в капельно по 10 000 ЕД каждые 4 часа.При этом удалось достигнуть увеличения времени свертывания крови до 18-23 минуты. На следующий день у б-ного диагностирова на правосторонняя нижнедолевая пневмония.Назначена натриевая соль бензилпенициллина (по 1000 000 ЕД каждые 4 ч) в/в.Через 4 ч время свертывания крови составило 8 минут. Какова ваша тактика?
Ответ: Изменить путь введения пенициллина

92. проведена радикальная операция по поводу рака желудка. На 4 сутки после операции при исследовании коагулограммы выявлены гиперкоагуляция и снижение фибринолитической активности крови.Целесообразно ли назначение антикоагулянтов?
Ответ: Антикоагулянты показаны,однако необходим тщательный контроль для предупреждения геморрагического синдрома

93. поступила в стац.с жал-ми на резкую слабость, одышку при ходьбе. При обследовании в анализе крови выявлена анемия (гемоглобин-56 г/л), цветной показатель 1,2,при осмотре языка-глоссит.В пунктате костного мозга выявлен мегалобластный тип кроветворения.Концентрация железа в сыворотке крови в пределах нормы.Диагноз: В 12-дефицитная анамия.Выберите наиболее оптимальный вариант лечения.
Ответ: Вит.В12 в дозе 500 мкг/сут через день, фолиевая кислота в дозе 1,5 мг/сут, сульфат железа(80 мгFe2+) 1 раз в сутки

94. после переохлаждения возникли озноб,повышение Т.тела до 38,6С, кашель с отделением слизисто-гнойной мокроты,боли в правой половине грудной клетки.Клинически и рентгенологически установлен диагноз правосторонней нижнедолевой пневмонии.Б-ной назначено лечение:цефазолин по 0,5 г 2р.в сутки вм,гемодез 400 мл в/в капельно, отхаркивающая микстура по 1ст.л.6р.в сутки.Выберите антиоксидантный препарат,наиболее
эффективно влияющий на процессы свободнорадикального окисления в легких,который следует добавить к проводимой терапии
Ответ: Вит.С

95. по поводу острого нарушения мозгового кровообращения по ишемическому типу давностью 12ч получает реополиглюкин по 400мл в/в капельно
1р.в сутки.Выберите наиболее эффективный в этой ситуации препарат,обладающий антиоксидантными свойствами
Ответ: Церебролизин

96. в течение 5 лет страдает деформирующим остеоартрозом нижних конечностей с выраженными синовитами. В анамнезе отмечает лекарственную
аллергию (на бутадион,гепарин,метиндол,пенициллин,теофиллин).В стаци-онаре б-ной назначены реопирин по 5 мл в/м 1 раз в сутки,гидрокортизона гемисукцинат по 100 мг в полость коленных суставов,тавегил по 0,001 г 2р.в сутки.Через 3 дня у б-ной появились зудящие эритематозные высыпания на коже туловища.Какова наиболее вероятная причина ухудшения состояния?
Ответ: Лекарственная аллергическая реакция

97. подтвержден диагноз ревматоидного артрита.Какие базисные средства для лечения ревматоидного артрита вы можете назначить: а)4,7-хлорхинолоновые препараты (делагил), б) Цитостатики (азатиоприн, циклофосфан и др.), в) Глюкокортикоиды (преднизолон), г) НПВС,
д) Препараты золота (кризанол), е) Салазопиридазин,
ж) Антибиотики (тетрациклины), з) D-пеницилламин,
и) Иммуномодуляторы (левамизол)
Ответ: а,б,д,е,з,и

98. Пациенту с ревматоидным артритом был назначен метотрексат. Как скоро проявится действие метотрексата?
Ответ: Через несколько месяцев

99. с ревматоидным артритом был назначен метотрексат. Какие вы предпримите меры по контролю за безопасностью фармакотерапии метотрексатом у данного пациента: а) Еженедельное проведение общего анализа крови
(лучше 2р.в неделю), б) Проведение анализа крови с определением количества тромбоцитов каждые 3-4 нед,
в) Проведение общего анализа мочи, г)Определение содержания мочевой кислоты, д) Проведение пробы на скрытую кровь в кале, е) Определение содержания трансаминаз,общего билирубина каждые 6-8 нед
Ответ: а,б,в,д,е

100. по поводу ревматизма длительно получает делагил.Какие вы предпримите меры по контролю безопасности терапии делагилом при его длительном применении: а)Общий анализ крови, б)Общий анализ мочи, в)ЭКГ, г)Исследование глазного дна, д)Исследование полей зрения, е)Рентгенологическое исследование органов грудной клетки, ж)Определение количества тромбоцитов, з) Исследование роговицы
Ответ: а,б,в,г,д,ж,з

101. 39 лет ревматоидный артрит, преимущественно суставная форма,2 степени активности.Какие варианты комбинированной терапии целесообразно назначить данному пациенту?
Ответ: Делагил по 0,25 г 3 раза в сутки, преднизолон по 15 мг/сут,кризанол в/м по 1 мл 5% раствора 1 раз в неделю

102. 63 лет страдает сахарным диабетом,принимает глибенкламид.Поступила в отделение с картиной острой правосторонней нижнедолевой пневмонии, подтвержденной рентгенологически.Был назначен хлорамфеникол,на который у б-ной отмечалась аллергическая реакция.Препарат отменили,выбран другой антибиотик-цефтриаксон.Однако при обследовании у б-ной выявлен низкий уровень клиренса креатинина (24 мл/мин),вследствие чего цефтриаксон был отменен.Каким препаратом следует продолжить лечение?
Ответ: Ципрофлоксацин

103. поступила в отделение с ж-ми на боли в правой молочной железе, повышение Т.до 39,5 С.Заболела 3 дня назад,на 10-й день после родов. При поступлении в отделение в верхненаружном квадранте правой молоч-
ной железы обнаружена гиперемия кожи,массивный инфильтрат с флуктуацией в центре.Диагноз:острый правосторонний мастит.Б-ная оперирована. Взят посев отделяемого раны.Определите антибиотик первого выбора
Ответ: Цефазолин

104. поступила в отделение с картиной острого правостороннего мастита.Заболела 3 дня назад,на 10-й день после родов. Б-ная оперирована.
Был назначен цефазолин.После 2-й инъекции препарата через 20 мин появились снижение АД, головокружение, тошнота, рвота, непроизвольное мочеиспускание, судорожный синдром. Какое осложнение развилось у пациентки?
Ответ: Анафилактическая реакция

105. Б-ная 21г.поступила в отделение с картиной острого правостороннего мастита.Заболела 3 дня назад, на 10-й день после родов.Б-ная опери рована.На цефазолин у б-ной отмечалась анафилактическая реакция,препарат был немедленно отменен.При посеве отделяемого раны выделены
стафилококк,образующий пенициллиназу,и гемофильная палочка.Выберите антибактериальный препарат с учетом бактериальной микрофлоры и особенностей фармакокинетики
Ответ: Левомицетин

106. страдает хроническим тонзиллитом и хроническим холециститом.В посеве отделяемого зева и в посеве желчи при обследовании выявлен золотистый стафилококк,образующий пенициллиназу.В анамнезе отмечена аллергия на оксациллин.Б-ной был назначен гентамицин.У б-ного клиренс креатинина составляет 50 мл/мин. Нужно ли корригировать режим дозирования препарата? Если да, то каким образом?
Ответ: Уменьшить кратность введения и снизить дозу

107. 50 лет обратилась с жалобами на общую слабость,жажду,частое мочеиспускание,зуд кожи и наружных половых органов.При осмотре выявлено ожирение (масса тела 96 кг при росте 168 см).Содержание глюкозы в крови 9,9 ммоль/л,в моче 1%, реакция на ацетон отрицательная. Какие гипогликемические препараты оптимальны в данном случае?
Ответ: Бигуаниды

108. 48 лет, поступил с жалобами на давящие боли, появляющиеся во время физической нагрузки, купирующиеся нитроглицерином. 3 года назад перенес инфаркт миокарда. В легких везикулярное дыхание. Тоны сердца приглушены, систолический шум на верхушке, частые экстрасистолы. ЧСС — 92 в мин. АД — 100/60 мм рт. ст. Печень не увеличена, отеков нет. ЭКГ — синусовая тахикардия, рубцовые изменения миокарда, частая желудочковая экстрасистолия. Назначен обзидан 160 мг/сутки, сустак-форте 19,2 мг/сутки, панангин, рибоксин.
Какой побочный эффект вероятен у больного при данной комбинации лекарственных средств?
Ответ: Гипотония, головокружение.

109. Больной М., 52 лет, поступил с жалобами на одышку, сердцебиение, боли в правом подреберье отеки на ногах. В течение 18 лет находится на диспансерном учете с диагнозом «ревматизм». Кожа бледная, акроцианоз, румянец щек. В базальных отделах легких-незвучные мелкопузырчатые хрипы. Границы относительной тупости средца расширены вверх и вправо. Тоны сердца приглушены, аритмичные, на верхушке систолический шум, акцент?? тона на легочной артерии. Пульс-96 в мин. ЧСС-140 в мин. АД — 130/85 мм рт. ст. Живот мягкий, печень на 3-4 см выступает из-под края реберной дуги. Отеки на ногах. Суточный диурез -650 мл. ЭКГ: отсутствует зубец Р, имеются волны «F-F», ритм неправильный. После в/в введения 10 мл 10% р-ра новокаинамида: восстановился синусовый ритм с ЧСС — 72 в мин, больному назначен новокаинамид внутрь по 0,5 г 4 раза в день, дигоксин 0,25 мг 1 табл.
3 раза в сутки, фуросемид 40 мг внутрь в течение 3-х дней. Через 5 дней у больного появилась тошнота, рвота, диарея, головокружение. ЭКГ: ритм синусовый,ЧСС-76 в мин, PQ -0,20 с, QRS-0,1с. Лечащий врач отменил дигоксин и фуросемид и назначил унитиол, препараты калия.
Оцените действия врача.
Ответ: Действия врача правильные, так как не только превышена средняя суточная доза дигоксина, но имеется и взаимодействие с новокаинамидом за связь с белком.

110. У больного Д., 53 лет, диагноз: ИБС, стабильная стенокардия III ФК, постинфарктный кардиосклероз, мерцательная аритмия,ХНК??Б ст. Принимал строфантин, дигоксин, фуросемид, панангин в средних терапевтических дозах. Неожиданно у больного повысилась температура до 38,4°С, появилась кашель, одышка, крепитация в легких справа. На рентгенограмме легких справа в нижней доле определяется участок инфильтрации. К лечению добавлены гентамицин, сульфокамфокаин,супрастин.
Возникновение каких побочных эффектов лечения наиболее вероятно у больного при такой комплексной терапии?
Ответ: При комбинации с фуросемидом наиболее вероятен нефротоксический эффект гентамицина.

111. Больной 28 лет поступил с жалобами на приступы сердцебиения, головную боль, озноб. Во время криза, который развивается 2-4 раза в году, повышается АД до 260/110 мм рт. ст., ЧСС-140 в мин., появляются бледность кожи, жгучие боли в области сердца, пульсация в голове, иногда повышение температуры тела до 38. После приступов полиурия. В межприступный период АД 120/80 мм рт. ст. При обьективном исследовании органической патологии со стороны внутренних органов не обнаружено. В анализах крови и мочи без патологии.
Укажите наиболее эффективный препарат (первый ряд) для купирования криза у больного:
Ответ: Фентоламин.

112. ревматоидным артритом на фоне курсового лечения метотрексатом возникло выраженное носовое кровотечение.Чем оно может быть обусловлено: а)Поражением сосудов носа вследствие основного патологического процесса, б)Повышением ПИ вследствие токсического гепатита,вызванного метотрексатом, в) Увеличением агрегации тромбоцитов под влиянием метотрексата,
г) Медикаментозно обусловленным снижением количества тро мбоцитов, д) Токсическим влиянием метотрексата на сосуды носа
Ответ: г,д

113. У больного К., 62 лет, АГ 1 степени. Последнее ухудшение состояния обусловлено психоэмоциональным стрессом. При осмотре: состояние относительно удовлетворительное, незначительная головная боль. АД-170/100 мм рт.ст. («рабочее» АД-120/70 мм рт.ст), ЧСС-90 в мин. Лечащий врач назначил анаприлин по 60мгсут, верапамил 160 мгсут.
Какие изменения Вы можете ожидать при назначении дополнительно к анаприлину верапамила?
Ответ: Усиление отрицательного дромотропного эффекта.

114. Больной С., 56 лет, по поводу стенокардии принимает нитросорбид (10 мг) 1т х 4р в день. Как изменится тактика проведения антиангинальной терапии, если у больного на фоне лечения нитратами возникнет мозговой инсульт?
Ответ: Отменить нитраты и назначить антиангинальный препарат из другой группы.

115. Больной 42 года с хроническим гломерулонефритом и артериальной гипертензией. При поступлении: АД 200/120 мм рт ст., пульс 75-80 уд в мин, отеки на лице, пояснице,голенях. Общий белок сыворотки крови 3.8 г%, в моче белок 16 г/л.. Укажите препараты, наиболее эффективные для гипотензивной терапии у данного больного:
Ответ: Эналаприл.

116. страдающему упорно рецидивирующим синдромом бронхиальной обструкции врач ввел п/к 1 мл адреналина.Какие проявления токсического
действия адреналина возможны в этой ситуации: а)Возбуждение ЦНС,б)Экстрасистолия, в)Токсическое поражение печени, г)Тахикардия, д)Блокада проведения импульса по проводящей системе сердца.
Ответ: а,б,г

117. 57 лет по поводу посинфарктного ардиосклероза,застойной сердечной недостаточности 2Б степени получает 40 мг фуросемида в/в и 300 мг
верошпирона внутрь.Какую диуретическую терапию Вы назначите б-ному в случае рефрактерности?
Ответ: Фуросемид 80 мг в/в и спиронолактон 300 мг внутрь

118. страдает неатопической бронхиальной астмой,сопровождающейся обильной бронхореей.Пульс 62 в мин.АД 140/80 мм.рт.ст. После назначения атропина сульфата у б-ной вначале отмечалось улучшение состояния-бронхорея резко уменьшилась,однако через 10 дней после начала лечения состояние вновь ухудшилось: появились лихорадка(37,8 С),одышка,кашель с трудно отделяемой мокротой, ЧСС 90 в минуту. Каковы причины подобных изменений состояния больной?
Ответ: Нарушение отхождения мокроты с ее последующим инфицированием

119. Женщина 52 лет страдает гипертонической болезнью?? ст. Принимает резерпин по 1 табл. (0.0001) 3 раза в день. АД нормализавалось через 1 неделю. Через 4 недели регулярного приема появились «голодные» боли в эпигастральной области, в ходе гастроскопии был диагностирован эрозивный дуоденит. Чем Вы обьясните его возникновение?
Ответ: Повышением тонуса n vagus на фоне резерпина и усилением желудочной секреции.

120. Больному 60 лет с ИБС, стабильной стенокардией IV ф.к. назначен кордарон 600 мг/сутки (в качестве антиангинального препарата).
Какие побочные эффекты могут наблюдаться у больного при длительном приеме кордарона?
Ответ: все перечисленное

121. При выборе режима дозирования лекарственных средств на основе Т?
определяют:
Ответ: кратность приема

122. Более точно характеризует скорость выведения лекарственных средств из организма:
Ответ: общий клиренс

123. Связь лекарственных средств с белками плазмы:
Ответ: определяет возможность развития побочных эффектов при сочетании лекарственных средств

124. Величина биодоступности важна для определения:
Ответ: пути введения лекарственных средств*

125. При длительном использовании сильных диуретиков может возникнуть:
Ответ: нарушение толерантности к глюкозе

126. Головокружение, отсутствие ощущения конечностей, затруднения при посадке и вставании без визуального контроля и другие симптомы токсического влияния встречаются у 75% пациентов, которые:
Ответ: получают стрептомицин

127. Передозировка симпатомиметиками вызывает:
Ответ: нарушения ритма

128. Побочные реакции, связанные с антибиотиком моксалактамом, включают в себе следующее:
Ответ: тромбоцитопению

129. Комбинированный прием индометацина и гентамицина наиболее часто вызывает:
Ответ: нарушение функции почек

130. Применение клавулановой кислоты в сочетании с амоксициллином позволяет:
Ответ: расширить спектр действия амоксициллина на штаммы бактерий, производящих бета-лактамазу

131. Больному, длительно получающему дифенин, планируется дополнительно назначить другой антиритмический препарат 1 класса, при назначении какого антиаритмика потребуется увеличение дозировки на 20-30% от стандартной?
Ответ: всех препаратов

132. Одновременный прием внутрь тетрациклина и препаратов Са2+ будет способствовать:
Ответ: снижению всасывания тетрациклина

133. Одновременно назначение хлорамфеникола и аценокумарола может привести:
Ответ: к снижению антибактериальной активности хлорамфеникола

134. При сердечной недостаточности:
Ответ: дофамин вызывает сужение сосудов коры почек в высоких дозах (более 10 мкг/кг/мин)

135. Для артериальной гипертензии характерно:
Ответ: увеличение концентрации натрия в сосудистой стенке

136. Апрессин (гидралазин):
Ответ: вызывает тахикардию

137. Бета-адреноблокаторы вызывают:
Ответ: снижение ЧСС

138. Справедливы такие утверждения об альфа-адреноблокаторах:
Ответ: все верно

139. Бета-1 — адреноблокаторы:
Ответ: избирательно действуют на бета1 -адренорецепторы, препараты безопасны при бронхиальной астме

140. Показаниями к применению бета-адреноблокаторов являются:
Ответ: нарушения сердечного ритма

141. Укажите правильные утверждения:
Ответ: строфантин в значительной степени разрушается в ЖКТ, в связи с чем прием его внутрь нерационален

142. Показания к назначению СГ:
Ответ: ХНК у больных ИБС, постинфарктным кардиосклерозом и постоянной формой мерцательной тахиаритмии

143. Фактор, повышающий риск развития интоксикации СГ:
Ответ: гипокалиемия

144. Состояние, повышающее риск развития интоксикации СГ:
Ответ: гипотиреоз

145. Для уменьшения риска развития толерантности к нитратам следует:
Ответ: делать перерывы между приемами препаратов

146. В случае развития толерантности к сустаку его можно заменить:
Ответ: корватоном

147. Головную боль может вызвать прием:
Ответ: верны ответы А,Б,В

148. Сходным по механизму действия с нитроглицерином является:
Ответ: молсидомин

149. При передозировке каких препаратов может возникнуть ортостатическая гипотония?
Ответ: нитратов

150. Назовите группу антиаритмиков, увеличивающих продолжительность потенциала действия:
Ответ: блокаторы калиевых каналов

151. Какой из перечисленных препаратов оказывает наиболее выраженное отрицательное инотропное действие?
Ответ: дизопирамид

152. Какие из некардиальных побочных эффектов характерны для большинства препаратов 1С класса?
Ответ: нарушение зрения

153. При терапии дизопирамидом может обостриться следующее заболевание:
Ответ: доброкачественная гиперплазия простаты с нарушением мочеиспускания

154. В каких случаях дозировка лидокаина должна быть изменена по сравнению со стандартной?
Ответ: у пациентов с печеночной недостаточностью

155. Укажите методы контроля за эффективностью применения диуретиков при отечном синдроме:
Ответ: все верно

156. Укажите методы контроля за безопасностью применения диуретиков при отечном синдроме:
Ответ: все верно

157. Укажите эффективный и безопасный способ пополнения запасов калия в организме:
Ответ: назначение панангина внутрь по 2 таблетки 3 раза в день

158. Укажите факторы риска возникновения побочных эффектов «петлевых» диуретиков:
Ответ: суточный диурез более 3 л после введения диуретика

159. Укажите начало действия спиронолактона:
Ответ: 4-5 дней

160. Для ощелачивания мочи используют следующие методы, кроме:
Ответ: цитрат калия 3 мг каждые 6 часов

161. Реакция мочи может быть кислой при использовании следующих препаратов, кроме:
Ответ: метионин

162. Отметьте ошибочные положения:
Ответ: нет ошибочных положенией

163. Распределите указанные препараты по степени кумуляции:
Ответ: неодикумарин

164. Отберите утвеждения, которые являются полностью правильными для препарата стрептокиназы:
Ответ: все верно

165. Отберите фактор, вызывающий тромбоз или способствующий тромообразованию:
Ответ: все верно

166. Какой из следующих лекарственных препаратов может уменьшить эффект антикоагулянтов непрямого действия?
Ответ: рифампицин

167. Какие побочные эффекты могут встречаться при применении гепарина?
Ответ: все перечисленное

168. Больному с бронхиальной астмой, длительно получавшему пролонгированные теофиллины, в связи с развитием инфекции мочевыводящих путей назначают ципрофлоксации. В этом случае необходимо:
Ответ: уменьшить дозу теофиллинов на 30%

169. У ребенка, длительно получающего карбамазепин в связи с наличием эпилепсии, развивается бронхообструктивный синдром с дыхательной недостаточностью 2-й ст. При назначении аминофиллина такому пациенту:
Ответ: доза аминофиллина должна быть увеличена в 1.5 раза

170. При назначении теофиллина курильщику:
Ответ: доза должна быть увеличена

171. Укажите препарат, снижающий элиминацию теофиллина при одновременном назначении:
Ответ: циметидин

172. У больному бронхиальной астмой, длительно получавшего теотард, на фоне гриппозной инфекции и лихорадки появилась тошнота, рвота, головные боли, бессонница. Менингиальные симптомы отрицательны. Терапевтическая тактика в данном случае:
Ответ: отменить теотард или снизить его дозу на 50%

173. Побочные эффекты теофиллина могут включать следующие явления, кроме:
Ответ: развития отечного синдрома

174. Специфическим побочным эффектом, возникающим при применении теофиллина у детей 1-го года жизни, является:
Ответ: мелена

175. Укажите ингаляционный глюкокортикостероидный препарат, обладающий наименьшей биодоступностью:
Ответ: флутиказона пропионат

176. Обозначьте ингаляционный глюкокортикостероидный препарат, обладающий наименьшим сродством к глюкокортикостероидным рецепторам легких человека:
Ответ: флутиказона пропионат

177. Укажите препарат, обладающий наибольшей степенью безопасности (по индексу безопасности:)
Ответ: преднизолон

178. Какой из глюкокортикостероидных препаратов в наибольшей мере способствует развитию миопатии?
Ответ: триамцинолон

179. Замедление выделения из организма натрия и воды, усиление выведения калия (минералокортикоидный эффект) в большей степени характерна:
Ответ: гидрокортизону

180. Минералокортикоидная активность отсутствует у:
Ответ: дексаметазона

181. Выберите правильный ответ. Глюкокортикоиды:
Ответ: являются контринсулярными гормонами

182. При проведении пульс-терапии более предпочтителен:
Ответ: метилпреднизолон

183. При назначении на длительный срок предпочтительнее использовать:
Ответ: преднизолон

184. Какой блокатор Н1-гистаминорецепторов противопоказан при анафилактическом шоке?
Ответ: дифенгидрамин (димедрол)

185. Выберите оптимальный блокатор Н1-гистаминорецепторов для лечения аллергического ринита:
Ответ: азеластин (аллергодил)

186. Обозначьте лекарственное средство из группы стабилизаторов мембран тучных клеток в лекарственной форме в виде порошка для ингаляций:
Ответ: кромоглициевая кислота (бикромат)

187. К иммуностимулятору микробного происхождения относится:
Ответ: рибомунил

188. Основное показание для назначения рибомунила заключается в:
Ответ: профилактике рецидивирующих инфекций верхних дыхательных путей

189. Хорошо проникают через гематоэнцефалический барьер следующие антибактериальные препараты:
Ответ: цефалоспорины III генерации

190. Новое поколение макролидных антибиотиков имеет следующие преимущества, кроме:
Ответ: почечный путь экскреции

191. Фторхинолоны отличаются от хинолонов следующими свойствами, кроме:
Ответ: бактериостатическим действием

192. Отметьте, какие утверждения в отношении цефалоспоринов правильны:
Ответ: все верно

193. Последствия приема антибиотиков включают:
Ответ: все верно

194. Укажите препарат выбора при инфекции мочевых путей, вызванной синегнойной палочкой:
Ответ: цефтазидим

195. Какие препараты показаны для лечения хламидийной инфекции мочеполового тракта:
Ответ: ровамицин

196. Укажите препарат с наименее выгодными фармакокинетическими характеристиками:
Ответ: кетоконазол

197. Укажите антимикотический препарат, не подвергающийся метаболизму в печени:
Ответ: флуконазол

198. Укажите антимикотический препарат (из группы аллиламинов), применяемый в первую очередь для лечения дерматомикозов:
Ответ: тербинафин

199. Укажите клиническое состояние, являющееся показанием для проведения монотерапии НПВС:
Ответ: внесуставные ревматические заболевания (миозит, тендовагинит, синовит)

200. Для ацетилсалициловой кислоты характерно:
Ответ: при оральном приеме всасывается в основном из верхнего отдела тонкого кишечника

201. В сравнении с индометацином у ацетилсалициловой кислоты более выражено:
Ответ: антиагрегантное действие на тромбоциты

202. На скорость выведения ацетилсалициловой кислоты и ее метаболитов влияют:
Ответ: уровень рН мочи

203. Желудочно-кишечные осложнения при применении ацетилсалициловой кислоты связаны с:
Ответ: все перечисленное

204. Для фенилбутазона характерно:
Ответ: все верно

205. При взаимодействии индометацина с другими препаратами:
Ответ: уменьшается диуретическая активность фуросемида

206. Какие побочные реакции НПВС коррегирует комплексный препарат артротек (диклофенак натрия + мизопростол)
Ответ: НПВС-гастропатии

207. Какие характеристики парацетамола выдвинули этот препарат на первое место в ряду анальгетиков-антипиретиков?
Ответ: более раннее наступление анальгетического и жаропонижающего эффекта

208. Выберите препарат, селективно ингибирующий циклооксигеназу2:
Ответ: мелоксикам

209. Наилучший анальгезирующий эффект фентанила наблюдается в комбинации с:
Ответ: дроперидолом

210. Назовите противовоспалительное средство пролонгированного действия:
Ответ: пироксикам

211. У Б-ой 52л.развилась картина гипертонического криза 2 типа с ЧСС 62 в мин.АД 200/140 мм.рт.ст.В легких большое кол-во влажных мелкопузырчатых хрипов.С какого препарата необходимо начинать купирование криза:
Ответ: Фуросемид

212. в течение 6 лет страдает сахарным диабетом 1 типа, получает инсулин по 54 ЕД/сут,что поддерживает уровень гликемии в пределах 7,0 ммоль/л.В последнее время в связи с повышением АД до 16090 мм.рт.ст. лечащий врач назначил гипотиазид в суточной дозе 75 мг в комбинации с эналаприлом в дозе 5 мг.Через 10 дней у больного уровень сахара в крови 10,5 ммольл, отмечается ухудшение самочувствия. Что является ведущей причиной изменения уровня сахара в крови?
Ответ: Комбинация эналаприла с гипотиазидом

213. развилась судорожная форма гипертонического криза, состояние тяжелое, цифры АД 200120 мм.рт.ст., ЧСС 120 в мин. С назначения какого препарата необходимо начинать терапию?
Ответ: Диазепама

214. на фоне применения антибиотика цефтриаксона в течение 10 дней развилась картина псевдомембранозного колита. Какой первый шаг алгоритма врачебной помощи?
Ответ: отмена цефтриаксона, назначение ванкомицина или метронидазола

215. по поводу обострения язвенной болезни желудка в комплексе терапии назначен кларитромицин. Назовите основные отличительные признаки препарата от эритромицина.
Ответ: все верно

216. после перенесенной операции на брюшной полости на 4-е сутки развилась левосторонняя нижнедолевая пневмония. Результаты экспресс-анализа показали наличие MRSA, пенициллино- и аминогликозидорезистентных штаммов энтерококков. Препараты выбора:
Ответ: Ванкомицин

217. находится в отделении интенсивной терапии по поводу синегнойной инфекции. Выберите препараты 1 ряда для лечения?
Ответ: Цефтазидим + аминогликозиды

218. 40 лет без сопутствующих заболеваний по поводу внебольничной пневмонии в амбулаторных условиях был назначен спирамицин внутрь по 3 млн. МЕ 2 рс, на 2-е сутки лечения отмечались интенсивные гастралгии, тошнота, однократная рвота. Выберите альтернативный препарат.
Ответ: доксициклин

219. с хроническим обструктивным бронхитом выявлена пневмония средней тяжести, в амбулаторных условиях назначен амоксиклав внутрь по 625 мг 3 рс. На 2-е сутки у больного развилась крапивница, бронхоспазм. Назовите альтернативный препарат для лечения пневмонии.
Ответ: моксифлоксацин внутрь

220. У ВИЧ- инфицированного б.44 лет диагностирована пневмоцистная пневмония. Назовите препарат для лечения?
Ответ: ко-тримоксазол вв 20 мгкгс 4 рс в течение 21 сут

221. У б-ной 28л. отмечаются ежедневные симптомы бронхиальной астмы, частые обострения, частые ночные симптомы, диагностирована тяжелая персистирующая бронхиальная астма. Назовите препараты базисной терапии.
Ответ: ингаляционные глюкокортикоиды (больше 1000 мкг беклометазона дипропионата) + ингаляционные бета-2-агонисты длительного действия

222. К врачу обратилась беременная женщина (срок беременности 6-7 нед.) с клиническими признаками острой пневмонии. Какие группы антибактериальных препаратов разрешены к применению у беременных?
Ответ: цефалоспорины

223. 57 лет по поводу артериальной гипертензии умеренной степени получает в монорежиме ингибитор АПФ — эналаприл. У больного через 2 года приема препарата отмечается недостаточность эффекта. Какой наиболее приемлемый вариант оптимизации терапии?
Ответ: добавление к препарату диуретика (гипотиазида или индапамида)

224. получает антибактериальный препарат по поводу инфекционного процесса. При внутривенной инфузии препарата отмечается реакция в виде выраженного покраснения кожных покровов верхней половины туловища, лица, шеи, симптомы значительно уменьшаются при снижении скорости инфузии. На какой препарат отмечается такая реакция?
Ответ: Ванкомицин

225. У беременной женщины отмечается активация ревматического процесса. Какой препарат из группы антикоагулянтов можно назначить беременной?
Ответ: Гепарин

226. поступил с ж-ми на повышение АД до цифр 15090 мм.рт.ст. на фоне психоэмоционального перенапряжения, сердцебиение, тревогу, нарушение сна. Год назад выявлен сахарный диабет 2 типа, получает манинил. Назовите препарат выбора для лечения АГ.
Ответ: Атенолол

227. в связи с симптомами стенокардии и нарушения ритма назначены препараты: анаприлин 200 мгс и верапамил 240 мгс в течение длительного времени. Какие возможны побочные реакции?
Ответ: Развитие а-в блокад, брадикардия

228. Женщина 34 лет принимает эстрогенсодержащие контрацептивные препараты. Лечащий врач назначил доксициклин в дозе 200 мгс в течение 2 недель. Какое вероятное взаимодействие ожидается?
Ответ: Снижается эффект контрацепции

229. Анестезиолог при введении кетамина с профилактической целью назначил больному диазепам. Профилактика какого состояния проводится таким образом?
Ответ: Постнаркозных галлюцинаций

230. 46 лет поступил с картиной острого деструктивного аппендицита. Препарат выбора для антибиотикопрофилактики?
Ответ: Цефазолин

231. К врачу обратился больной с жалобами на кашель, повышение температуры до 39 С, боли в грудной клетке. Диагностирована правостронняя бронхопневмония. Назначен препарат в течение 3 суток, обладающий постантибиотическим эффектом. Назовите препарат выбора.
Ответ: Азитромицин

232. по поводу острого правостороннего пиелонефрита назначен цефазолин по 2 гс в течение 10 дней. Назовите наиболее типичную ошибку при выборе данного антибиотика
Ответ: Недостаточно высокая активность в отношении грамотрицательной флоры

233. одновременно получает в течение 14 дней фторхинолоновый антибиотик — офлоксацин по поводу инфекции мочевыводящих путей и диклофенак натрия в связи с суставным синдромом. Какое вероятное взаимодействие ожидается?
Ответ: повышение риска возбуждения ЦНС и развития судорог

234. после переохлаждения возникли озноб,повышение Т.тела до 38,6С, кашель с отделением слизисто-гнойной мокроты,боли в левой половине грудной клетки.Клинически и рентгенологически установлен диагноз левосторонней нижнедолевой пневмонии.Б-ной назначено лечение:цефазолин по 1 г 2р.в сутки вм,гемодез 400 мл в/в капельно, отхаркивающая микстура по 1ст.л.6р.в сутки. На 3-е сутки отмечалась реакция в виде крапивницы, кожного зуда. Выберите антибактериальный препарат для замены?
Ответ: Спирамицин

235. При проведении наркоза анестезиолог с целью антибиотикопрофилактики назначил антибактериальный препарат. У больного развилась остановка дыхания. Какой препарат использовался?
Ответ: Гентамицин

236. в течение 10 лет страдает деформирующим остеоартрозом нижних конечностей с выраженными синовитами. В анамнезе отмечает лекарственную аллергию на бутадион. В отделении б-ной назначены реопирин по 5 мл в/м 1 раз в сутки.Через день у б-ной появились зудящие эритематозные высыпания на коже туловища.Какова наиболее вероятная причина ухудшения состояния?
Ответ: Лекарственная аллергическая реакция

237. по поводу нарушения ритма сердца назначен новокаинамид, одновременно в связи с сезонным аллергическим ринитом назначен цетиризин. Какое вероятное взаимодействие ожидается?
Ответ: тяжелые формы аритмии (типа пируэта)

238. Пациенту с СКВ был назначен метотрексат. Через какое время ожидается стабильное проявление лечебного эффекта?
Ответ: бірнеше айдан кейін

239. с гипертоническим кризом был назначен препарат фозиноприл. Несмотря на прием адекватной дозы препарата не отмечено снижение уровня АД в ближайшие минуты и часы. Назовите причину.
Ответ: При кризах не используются депо-препараты

240. с целью плановой терапии АГ был назначен апрессин в течение длительного времени. У больного через месяц применения препарата стали отмечаться сердцебиение, стенокардитические боли, снижение эффекта от лечения. Назовите основную причину развившихся явлений
Ответ: Апрессин не назначается для плановой терапии АГ

241. 42 лет поступил в отделение интенсивной терапии по поводу тяжелого гипертонического криза. В течение 5 суток внутривенно инфузионно вводился нитропруссид натрия. На 6-е сутки у больного развилась картина интоксикации в виде неукротимой рвоты, снижения функции сердечно-сосудистой, дыхательной и выделительной систем. Назовите основную причину развившегося состояния.
Ответ: Передозировка препарата (накопление тиоцианатов в крови)

242. 54 лет страдает сахарным диабетом 1 типа,принимает пролонгированный инсулин.Поступила в отделение с картиной острой правосторонней нижнедолевой пневмонии, подтвержденной рентгенологически.Был назначен цефтриаксон,на который у б-ной отмечалась аллергическая реакция.Препарат отменили,выбран другой антибиотик- ципрофлоксацин в сочетании с амикацином.Однако при обследовании у б-ной выявлен низкий уровень клиренса креатинина (30 мл/мин),вследствие чего амикацин был отменен.Каким препаратом следует продолжить лечение?
Ответ: Спирамицин

243. поступила в отделение с ж-ми на боли в низу живота, повышение Т.до 39,5 С.Заболела 2 дня назад,на 6-й день после родов. При гинекологическом осмотре — картина острого послеродового эндометрита. Результаты бакпосева: золотистый стафилококк, образующий пенициллиназу, протей. Определите антибиотик первого выбора
Ответ: Цефепим

244. 25 л.поступила в отделение с картиной острого правостороннего пиелонефрита.Заболела 3 дня назад после переохлаждения. Был назначен цефазолин.После 2-й инъекции препарата через 10 мин появились снижение АД, головокружение, тошнота, рвота, непроизвольное мочеиспускание, судорожный синдром. Какое осложнение развилось у пациентки?
Ответ: Анафилактическая реакция

245. Б-ная 28 л.поступила в отделение с картиной острого правостороннего мастита.Заболела на 12-й день после родов.Б-ная оперирована.На цефазолин у б-ной отмечалась анафилактическая реакция,препарат был немедленно отменен.При посеве отделяемого раны выделены стафилококк,образующий пенициллиназу,и кандиды.Выберите антибактериальный препарат с учетом бактериальной микрофлоры и особенностей фармакокинетики
Ответ: Оксациллин + флуконазол

246. страдает хроническим холециститом.В посеве желчи при обследовании выявлены золотистый стафилококк и кишечная палочка. В анамнезе отмечена аллергия на оксациллин. Препараты выбора.
Ответ: Цефтриаксон

247. 58 лет обратилась с жалобами на общую слабость,жажду,частое мочеиспускание,зуд кожи и наружных половых органов.При осмотре: масса тела 56 кг при росте 168 см).Содержание глюкозы в крови 12,3 ммоль/л,в моче 1,5%, реакция на ацетон отрицательная. Какие гипогликемические препараты оптимальны в данном случае?
Ответ: Препараты сульфонилмочевины

248. 53 лет, поступил с жалобами на сердцебиение, перебои, временами одышку. Эти явления стали беспокоить после перенесенного инфаркта миокарда 2 года назад. Прием новокаинамида в течение 3 месяцев принес значительное облегчение. Однако в последнее время самочувствие ухудшилось. Дальнейшая тактика ведения больного.
Ответ: полное обследование и выбор препарата

249. 33 лет по поводу послеоперационного гнойного перитонита получает комбинацию препаратов: цефтриаксон + амикацин + метронидазол. В анамнезе — желчнокаменная болезнь. Назовите препарат, который не должен назначаться больному.
Ответ: Цефтриаксон

250. 45 лет по поводу кандидозной пневмонии был назначен флуконазол вв в течение 3 дней, затем внутрь. На 4-е сутки лечения отмечена реакция в виде сильных головных болей, тошноты. Решено было заменить препарат на кетоконазол. Оцените адекватность тактики.
Ответ: Кетоконазол не является адекватной заменой в силу невыгодных фармакокинетических характеристик

251. 42 лет, поступил с жалобами на выраженное сердцебиение, повышение цифр АД до 240140 мм.рт.ст. При обследовании выявлено значительное повышение в крови уровня катехоламинов. Назовите препарат выбора для купирования криза.
Ответ: фентоламин

252. 50 лет, поступил с картиной острого левостороннего пиелонефрита. Были назначены цефазолин + гентамицин в среднетерапевтических дозах. У больного при обследовании клиренс креатинина — 50 млмин. Какие вероятные последствия терапии?
Ответ: Риск нефротоксичности

253. 48 лет с пароксизмальной над- и желудочковой тахикардией был назначен кордарон вв в первые сутки, затем внутрь. При обследовании выявлено нарушение функции щитовидной железы, а-в блокада 2-3 степени. Дальнейшая тактика.
Ответ: Отмена препарата, назначение новокаинамида

254. 50 лет находится в отделении кардиореанимации по поводу острого инфаркта миокарда, получает комплекс терапии. Какие основные параметры мониторинга при назначении прямых антикоагулянтов?
Ответ: АЧТВ, время свертывания крови, моча на эритроциты

255. В отделении находится больной после операции по поводу аппендицита. Лечащий врач назначил гентамицин по 80 мг 3 рс вм. Пациент 2 года назад перенес острый гломерулонефрит, в настоящее время клиренс креатинина составляет 50 млмин. Какая коррекция требуется?
Ответ: замена на препарат, не обладающий нефротоксичностью

256. 56 лет, получает дигоксин по 0,25 гс в течение последнего года. В настоящее время стало отмечаться повышение цифр АД до 180110 мм.рт.ст. Лечащим врачом назначен лизиноприл в дозе 10 мгс. Через 3 месяца у пациента диагностирована дигиталисная интоксикация. Дальнейшая тактика.
Ответ: назначение гипотензивного препарата другого ряда

257. 53 лет диагностирован кандидозный и аспергиллезный менингит. Препараты выбора.
Ответ: Амфотерицин В

258. 58 лет, получающего длительное время глюкокортикоиды в комплекс терапии был включен синтетический антибиотик широкого спектра в связи с инфекцией репродуктивной сферы. На 14 сутки совместного применения отмечено тяжелое осложнение в виде разрыва ахиллова сухожилия. Назовите антибиотик, который в сочетании с ГКС вызвал данное осложнение.
Ответ: Левофлоксацин

259. в отделение поступил Б.43 лет с картиной острой хламидийной пневмонии. Назовите препараты выбора.
Ответ: Ровамицин

260. У больного 24 лет диагностирована неосложненная форма гонореи. Назовите препарат выбора.
Ответ: Цефтриаксон

261. Выберите правильное утверждение: а)биодоступность-количество ЛС,поступающее в системный кровоток,выраженное в процентах от введенной дозы,б)биодоступность определяется величиной адсорбции ЛС в ЖКТ и выра-
женностью эффекта первого прохождения через печень.в)биодоступность определяют по формуле: F = AUC (в/м или внутрь)/AUC (в/в).г)биодоступность ЛС при внутримышечном введении определяется степенью его всасывания и биотрансформации в организме.
Ответ: а, б, в

262. страдает неатопической бронхиальной астмой,сопровождающейся обильной бронхореей.Пульс 62 в мин.АД 140/80 мм.рт.ст.Какие препараты более предпочтительны?
Ответ: Атровент

263. Механизмы всасывания ЛС в кишечнике:
а) пассивная диффузия, б) фильтрация, в) активный транспорт,г) облегченный транспорт, д) пиноцитоз:
Ответ: а, д

264. поступил с ж-ми на изжогу,боли в эпигастральной области натощак, купируемые приемом натрия гидрокарбоната.При ФЭГДС выявлена язва (0,5см в диаметре) в ампуле 12 п.к.РН-метрия желудочного сока: кислотообра-
зующая функция средней интенсивности с низкими щелочными резервами,холинергический тип рецепции. Диагноз: язвенная болезнь 12 п.к.в стадии обострения. Выберите наиболее эффективное и безопасное ЛС и определите режим его дозирования:
Ответ: Пирензепин до еды по 0,05 г 3 раза в сутки в течение 2дней, затем по 0,05 г 2 раза в сутки

265. выявлена дискенезия желчного пузыря по гипертоническому типу. Выберите оптимальный вариант лечения.
Ответ: Но-шпа по 1-2 табл.3 раза в сутки, отвар бессмертника по 1/2 стакана за 30 мин до еды

266. страдает хрон.холецистопанкреатитом в течение 5 лет.За последнюю неделю после нарушения диеты отмечает усиление болей в правом по дреберье,тошноту,горечь во рту.Выберите наиболее эффективные желчегонные средства, обладающие одновременно противомикробной активностью:
а) Аллохол, б) Холензим, в) Никодин, г) Отвар пижмы,
д) Ксилит
Ответ: а,в

267. с суицидальной целью выпила 20 таблеток феназепама.Через 2 ч после приема препарата доставлена в стационар. Б-я в сознании,но резко заторможена.Проведено промывание желудка.Выберите наиболее оптимальные слабительные средства: а) Глауберова соль, б) Сульфат магния, в) Экстракт коры крушины, г) Бисакодил,
д) Касторовое масло, е) Морская капуста, ж) Вазелиновое масло
Ответ: а,б,д

268. 46 лет поступил в отделение кардиореанимации с острым трансмуральным инфарктом миокарда, возникшим около 5 ч назад.Назначения:анаприлин 20 мг 4 раза в сутки внутрь, гепарин в/в капельно по 10 000 ЕД каждые 4 часа.При этом удалось достигнуть увеличения времени свертывания крови до 18-23 минуты. На 4-й день у б-ного выявлена микрогематурия (22 эритроцита в поле зрения). Какова ваша тактика?
Ответ: Снизить дозу гепарина вплоть до времени свертывания крови не менее 10-12 мин

Здравствуйте, дорогие друзья!

Вот и еще одна тема у нас с вами пройдена. Очень надеюсь, что я помогла вам разобраться с лекарственными .

Хотите проверить себя, все ли поняли? Все ли уложилось «по полочкам»?

В таком случае ниже будет тест, который я для вас приготовила.

Как обычно, постаралась сделать его максимально практичным.

В некоторых вопросах фигурируют препараты, которые мы с вами подробно не разбирали. Это я сделала специально. А кто сказал, что их знать не надо? 🙂

Если затрудняетесь, будет повод заглянуть в инструкцию.

Не расстраивайтесь, если результат окажется не очень...

Мне почему-то кажется, что здесь, на моем блоге, бывают только те трудяги фармбизнеса, которые хотят разобраться во всем досконально. Об этом говорят ваши вопросы в комментариях. Так что не опускайте рук и изучайте все, что я здесь выкладываю. Если нужно, проходите одну и ту же тему еще и еще раз.

Я уже говорила вам, что люблю тесты? Это классная форма проверки знаний, чтобы понять, над чем еще нужно поработать.

Мне по своей основной работе приходится их составлять часто.

Не могу сказать, что сотрудники компании, в которой тружусь, от них в восторге. Они всегда напрягаются, поскольку от результатов тестирования что-нибудь, да зависит.

Но здесь, на блоге, никаких нервотрепок не бывает. Его можно проходить хоть 100 раз, чтобы запомнить и понять, что к чему. Тем более, ограничений по времени никаких.

А значит, есть достаточно времени, чтобы подумать над вопросом и без спешки порассуждать.

Например, вот такой вопрос:

«Посетитель жалуется на сильный зуд, жжение в области ануса, иногда бывает кровь в стуле. Какой препарат Вы предложите в первую очередь?»

Ход мыслей будет примерно таким:

«Если сильный зуд и жжение, значит, нужно что-то с гормональным компонентом. Поэтому препараты X и Y отметаем.

Остается два. Если бывает кровь в стуле, значит, препарат с гепарином лучше не давать. Отметаем и препарат Z. Следовательно остается препарат...»

В общем, спокойненько прохОдите тест, видите неважнецкий результат, значит надо что-то еще раз почитать, а потом пройти снова и снова, пока результат вас удовлетворит.

Итак, в тесте 18 вопросов. В каждом возможен только ОДИН правильный вариант ответа. Максимальное количество баллов, которые можно набрать — 18.

Желаю удачи!

Как, друзья, Вам понравился этот тест? А свои результаты? 🙂

Расскажите, что у вас получилось! Было сложно?

Полный сборник тестов в формате «word» с ответами вы можете приобрести .

А я с вами прощаюсь до новой встречи на блоге « ».

С любовью к вам, Марина Кузнецова